Vous êtes sur la page 1sur 37

Independent learning program for GPs

Unit 539 June 2017

Infectious diseases

www.racgp.org.au/check
Disclaimer
The information set out in this publication is current at the date of first publication and is intended
for use as a guide of a general nature only and may or may not be relevant to particular patients
or circumstances. Nor is this publication exhaustive of the subject matter. Persons implementing
any recommendations contained in this publication must exercise their own independent skill or
judgement, or seek appropriate professional advice relevant to their own particular circumstances
when so doing. Compliance with any recommendations cannot of itself guarantee discharge of the
duty of care owed to patients and others coming into contact with the health professional and the
premises from which the health professional operates.

While the text is directed to health professionals possessing appropriate qualifications and skills in
ascertaining and discharging their professional (including legal) duties, it is not to be regarded as
clinical advice and, in particular, is no substitute for a full examination and consideration of medical
history in reaching a diagnosis and treatment based on accepted clinical practices.

Accordingly, The Royal Australian College of General Practitioners and its employees and agents
shall have no liability (including without limitation liability by reason of negligence) to any users of
the information contained in this publication for any loss or damage (consequential or otherwise),
cost or expense incurred or arising by reason of any person using or relying on the information
contained in this publication and whether caused by reason of any error, negligent act, omission or
misrepresentation in the information.

Subscriptions
For subscriptions and enquiries please call 1800 331 626 or email check@racgp.org.au

Published by
The Royal Australian College of General Practitioners
100 Wellington Parade
East Melbourne, Victoria 3002, Australia
Telephone 03 8699 0414
Facsimile 03 8699 0400
www.racgp.org.au

ABN 34 000 223 807


ISSN 0812-9630

The Royal Australian College of General Practitioners 2017

This work is subject to copyright. Unless permitted under the Copyright Act 1968, no part may be
reproduced in any way without The Royal Australian College of General Practitioners prior written
permission. Requests and enquiries should be sent to permissions@racgp.org.au
We recognise the traditional custodians of the land and sea on which we work and live.
Independent learning program for GPs

Independent learning program for GPs

Infectious diseases
Unit 539 June 2017

About this activity 2

Acronyms 3

Case 1 An outbreak of influenza virus 4

Case 2 Simona has genital pain 12

Case 3 David has an itchy rash 16

Case 4 Zoe has stomach cramps 21

Case 5 Mary-Joy has an ongoing cough 27

Multiple choice questions 33

The five domains of general practice


Communication skills and the patientdoctor relationship
Applied professional knowledge and skills
Population health and the context of general practice
Professional and ethical role
Organisational and legal dimensions
ABOUT THIS ACTIVITY check Infectious diseases

ABOUT THIS ACTIVITY Dr Burns is also RACGP NSW&ACT Disaster Response


Representative, RACGP representative on the GP Round Table,
Mortality related to communicable disease has decreased from 13% of
member of the NSW Mental Health Disaster Advisory Committee,
all deaths in 1907 to 1.3% in 2009.1 However, infectious diseases
Chair of the Oceania Chapter of the World Association of Disaster
remain a major health concern and, usually, patients with infections
Emergency Medicine, member of the Immunisation Coalition, RACGP
will first present to general practice. While some infectious diseases
representative on the National Immunisation Committee and chaired
resolve without intervention, others can be chronic and have a serious
the RACGP Pandemic flu kit taskforce. Dr Burns is currently
adverse impact on an individuals daily activities.
undertaking a PhD on the role GPs have to play in disasters and
As the speed and scale of national and international travel continue to pandemics at the Australian National University. She is a conjoint
increase, general practitioners need to be at the forefront of senior lecturer at the University of Western Sydney.
coordinated, strategic approaches to tackling pandemics.1 Genital
Winnie Chen (Case 5) BMedSci, MBBS, is a general practice registrar
herpes is caused by the Herpes simplex virus (HSV), and is a common
at Centre for Disease Control, Darwin. She is interested in medical
sexually transmissible infection (STI); an estimated one in eight
publishing and previously worked at the Medical Journal of Australia
Australians have the virus.2
as a deputy medical editor.
Scabies is a highly contagious parasitic skin infection caused by
Alvin Chong (Case 3) MBBS, MMed, FACD, is specialist dermatologist
Sarcoptes scabiei. Often, treatment needs to be targeted not only at
and senior lecturer at St Vincents Hospital Melbourne, Skin and
the patient, but also at close contacts, including household members.3
Cancer Foundation Inc and the University of Melbourne. He is also
Infectious gastroenteritis can be caused by bacteria, viruses or principal dermatologist at Ivanhoe Dermatology Clinic, Victoria. His
parasites, and usually occurs when the source is ingested.4 special interests are in dermatology education and in the
Tuberculosis is caused by Mycobacterium tuberculosis, and most dermatological care of immunosuppressed patients.
cases are found in those from countries of high prevalence.5
Rachel Davenport (Case 3) MBBS, is a research and education fellow
at the Skin and Cancer Foundation, Victoria. She is completing a
LEARNING OUTCOMES Masters of Medicine in Clinical Epidemiology. Dr Davenports research
At the end of this activity, participants will be able to: focus is on dermatological conditions in the immunocompromised
host. Her other special interest is vulval dermatology.
outline the role of the general practitioner in the event of a pandemic
Glynn Kelly (Case 1) MBBS (Hons), BSc, Cert Higher Ed, MMed,
discuss the diagnosis and management of genital herpes FRACGP, is a Brisbane-based general practitioner who has a special
describe the management of patients with scabies interest in disaster and pandemic planning, preparation and
responding particularly in the interface between primary care and
summarise the assessment and management of gastroenteritis and hospitals and other health services. Dr Kelly is the Chair, RACGP
its complications Disaster Network and is the Chief Medical Officer of St John Australia.
discuss the diagnosis and treatment of patients with tuberculosis. He also has a professorial appointment with the QUT Centre for
Emergency and Disaster Management.
AUTHORS Vicki Krause (Case 5) MD, DTM&H, FAFPHM, is director of Northern
Territory (NT) Centre for Disease Control and the NT Head of
Bruce Bartley (Case 4) MBBS, FRCSE, FACEM, is deputy director of
Tuberculosis/Leprosy Clinical and Program Services, based in
the emergency department at University Hospital Geelong. He has a
Darwin, NT and has edited the NT Disease Control Bulletin for the
surgical background and fulfils leadership and representative roles in
past 26 years.
trauma and disaster medicine at the local, regional, state and national
levels. Dr Bartley is chairman of the Barwon Health Trauma Audit Catriona Ooi (Case 2) FAChSHM, MM(HIV/STDs), MBBS BSc (Med),
Committee and has an ongoing leadership role in regional outreach DipFP, DipGov, is a sexual health physician and the clinical lead at the
and support including supervision of the regional and remote specialist Western Sydney Sexual Health Service. Dr Ooi teaches is a senior
training program at nearby Colac. His current priorities include lecturer at the University of Sydney, teaching in the clinical medical
emergency health systems improvement at a local and regional level, school in Westmead. She is involved in ongoing research and has
Aboriginal and Torres Strait Islander health, and completion of a published in textbooks and peer-reviewed journals.
Masters in Public Health and Tropical Medicine.
Kate Sandy (Case 1) MBBS, is a member of the RACGP Specific
Penny Burns (Cases 1 and 4) BMed, MPHTM, is a general practitioner Interests Disaster Management group. She is a graduate of the
with a special interest in travel and tropical medicine. She has worked in Australian Film, Television & Radio School and has been involved in
general practice and hospitals in Papua New Guinea and in Colombia disaster management education over the past five years. Dr Sandy
where gastroenteritis is a constant presentation. Dr Burns has a special has compiled and presented disaster scenarios for GPs, practice
interest in public health and infectious outbreaks and has completed a nurses and postgraduate students. She is currently an intern at
Masters of Public Health and Tropical Medicine at James Cook University. Nepean hospital.

2
check Infectious diseases ABOUT THIS ACTIVITY

Dean Zinghini (Case 4) MBBS, is a general practice registrar working REFERENCES


in Port Macquarie, New South Wales. Since completing university, he 1. Department of Health. National framework for communicable disease control.
has fulfilled both teaching and representative roles as a conjoint Canberra: DoH, 2014. Available at www.health.gov.au/internet/main/publishing.nsf/
lecturer with Western Sydney University and worked with quality Content/ohp-nat-frame-communic-disease-control.htm [Accessed 19 April 2017].
improvement on the patient care committee at Royal Prince Alfred 2. Sexual Health Australia. Genital herpes. Sydney: SHA, 2015. Available at www.
sexualhealthaustralia.com.au/genital_herpes.html [Accessed 19 April 2017].
Hospital. He has a special interest in infectious disease and 3. The Royal Childrens Hospital Melbourne. Scabies Symptoms and treatment.
paediatrics, completing his Diploma of Child Health in 2016. Parkville, Vic: RCH, 2010. Available at www.rch.org.au/kidsinfo/fact_sheets/
Scabies_symptoms_and_treatment [Accessed 19 April 2017].
4. The Royal Childrens Hospital Melbourne. Gastroenteritis. Parkville, Vic: RCH, 2015.
PEER REVIEWERS Available at www.rch.org.au/clinicalguide/guideline_index/Gastroenteritis [Accessed
19 April 2017].
Paul Griffin FRACP, FRCPA, FACTM, AFACHSM, MBBS, BSc (Hons), is 5. Department of Health. Mycobacterial infections (tuberculosis). Melbourne: DoH, 2015.
the Director of Infectious Diseases at Mater Health Services in Brisbane,
conjoint senior lecturer at the University of Queensland School of
Medicine, and affiliate senior lecturer at the Mater Research Institute. He
has fellowships in infectious diseases from The Royal Australasian
College of Physicians, and in Clinical Microbiology from The Royal
College of Pathologists of Australasia and the Australasian College of
Tropical Medicine. Concurrently, Dr Griffin maintains a joint appointment
as a scientist at QIMR Berghofer/Mater Medical Research Institute with a
primary research interest in tropical medicine particularly malaria human
challenge studies. He is the principal investigator and manager of
medical services at Qpharm Pty Ltd, a specialised contract research
organisation within QIMR Berghofer specialising in early and late phase
trials in infectious diseases where he has been the principal investigator
on in excess of 40 clinical trials predominantly in infectious diseases. As
a clinical microbiologist, Dr Griffin maintains an active interest in
diagnostic microbiology with a focus on the detection of antibiotic
resistance particularly VRE with MALDI-TOF MS.
Lawrence Tan MBBS, DCH, DRCOG, MPH, FRACGP is a part-time
general practitioner in Southwest Sydney, and senior lecturer in the
Department of General Practice, Western Sydney University. He enjoys
teaching medical students and registrars. Dr Tans research interests
are in cultural and linguistic diversity and how this affects the delivery
of primary healthcare.

ACRONYMS
ACRRM Australian College of Rural and GBS Guillain-Barr syndrome MSM men who have sex with men
Remote Medicine GORD gastro-oesophageal reflux disease NAAT nucleic acid amplification test
AFB acid-fast bacilli GP general practitioner NSAID non-steroidal anti-inflammatory drug
AGE acute gastroenteritis GPRT GP Round Table PCR polymerase chain reaction
AHPPC Australian Health Protection HBV hepatitis B virus PEP post-exposure prophylaxis
Principal Committee
HIV human immunodeficiency virus PHU public health unit
AMI acute myocardial infarction
HPV human papillomavirus PPE personal protective equipment
CMV cytomegalovirus
HSV Herpes simplex virus PPI proton pump inhibitors
COCP combined oral contraceptive pill
HSV-1 Herpes simplex 1 RACGP The Royal Australian College of
DOTS directly observed therapy short General Practitioners
course HSV-2 Herpes simplex 2
IGRA interferon gamma release assays STI sexually transmissible infection
ED emergency department
LTBI latent tuberculosis infection TB tuberculosis
EHEC enterohaemorrhagic Escherichia coli
MCS microscopy, culture and sensitivity WHO World Health Organization
ELISA enzyme-linked immunosorbent
assay MERS Middle East respiratory syndrome

3
CASE 1 check Infectious diseases

FURTHER INFORMATION
CASE 1
Over the past few weeks, during the development of a pandemic
plan for your practice, using The Royal Australian College of
AN OUTBREAK OF INFLUENZA VIRUS General Practitioners (RACGPs) Pandemic flu kit, you have kept an
You are working in a small urban general practice in eye on the news about the situation in Indonesia. Reports indicate
western Sydney with four other GPs. It is January and the HzNz virus is spreading in Indonesia, with small numbers of
you are busy managing an unusual peak in asthma cases being reported in neighbouring countries.
following recent local bushfires. You have yet to even
The practice manager draws your attention to an increase in the
think about the next flu season.
number of patients who are contacting the surgery, concerned
The evening news reports an outbreak of a new strain of about the overseas situation and what it means for them. An
influenza A virus, HzNz, in Indonesia. It is thought to increasing number of patients have been presenting with
have originated in a rural poultry-farming region where symptoms they think might be the HzNz.
people live closely with their chickens. The outbreak has
reached the edge of the nearest city in Indonesia and is QUESTION 3
spreading rapidly. Thousands of people have developed
an influenza-like illness and more than 100 deaths have Is this now a pandemic? When does it become a pandemic?
been recorded so far. In an attempt to slow the spread,
the Indonesian government has closed schools and
public gatherings have been cancelled.

QUESTION 1

Why is an overseas outbreak of relevance to individual general


practitioners (GPs) in Australia? Why is a rapidly spreading influenza A
outbreak of particular concern?

QUESTION 4

What are the pandemic stages? Which stage of a pandemic is


Australia in at this point in this scenario?

QUESTION 2

Where will you look for relevant, accurate information updates?

4
check Infectious diseases CASE 1

QUESTION 5 been investigated in Australia over the past few weeks, all in
returning travellers from Indonesia, but not in the state where you
How would you prepare for this potential impending pandemic in your work. All have been negative for HzNz.
practice in Australia (eg maintain infection prevention and control,
business continuity, and communication with staff and patients)? QUESTION 6

What stage have we moved into now (refer to Figure 1)?

FURTHER INFORMATION
It is now six weeks since the initial reports of the HzNz outbreak
overseas (late February). A small number of possible cases have

Figure 1. Example outline of message that might be received at this stage from the Department of Health through
the GPRT through GP organisations
Update on Department of Healths response to Influenza A HzNz

What is new:
There are currently no confirmed cases of HzNz in Australia however GPs are asked to be alert for potential presentations.
Clinical case definition:
Sudden onset of fever 38oC PLUS
Cough and/or haemoptysis PLUS
Purpuric rash OR
Somnambulism (sleepwalking)

AND

Epidemiological evidence:
Recent travel to Indonesia within two weeks of symptom onset OR Contact with a confirmed case

Specific action for GPs:


Isolate and report any probable cases immediately to the local public health unit.
Investigation with nasopharyngeal swab using full personal protective equipment is recommended in all probable cases who meet the case definition.
Antivirals (for five days) are recommended for all probable and confirmed cases if <48 hours symptoms onset.
Antiviral prophylaxis is not currently recommended.
Those thought to be at higher risk include: children, elderly, pregnant women and those with chronic disease.

What we do not know:


Case fatality and prognosis is unknown. Notifications to PHUs will help collect this information.
Work is commencing immediately on the development of a vaccine which may not be available for three to four months.

Further information: Further details and updates will be available at www.hznzhznz.com.au

5
CASE 1 check Infectious diseases

FURTHER INFORMATION FURTHER INFORMATION


Just as you are reading the information, your practice manager Joshs test results returns and he is positive for HzNz.
rushes into your room to report that Jessica, 38 years of age and
Note: Australia is now in Response Initial action stage with the
one of your regular patients, has just returned from Bali with her
novel strain, HzNz now detected in Australia. During this stage you
son, Josh, aged 14 years, who has been coughing since yesterday,
are actively looking for HzNz cases and managing them as you did
walked in his sleep last night, and is now feeling hot (temperature
Josh.
on examination 38.5C). The practice nurse has immediately
isolated them in an empty consultation room (if there were no A further four to six weeks pass and there has been a significant
empty consultation rooms available, she would have asked them to spread of the HzNz influenza. Despite the prompt reporting and
stay in their car until you were ready to see them) and asked them management of early cases by local GPs, yourself included, more
to wear masks. than 1000 cases have been confirmed in your state alone, and
significant spread is occurring across Australia. Updated
QUESTION 7 information comes out from Department of Health regarding case
definition and testing recommendations (Figure 2).
What features of Joshs history are you most interested in?
Figure 2. Example of a message that might be
received at the later stage of a pandemic
Update on Department of Healths response to Influenza A HzNz

Key points for GPs


1000 confirmed cases HzNz in NSW with significant spread to Vic
and Qld
Symptoms of HzNz Nearly all cases of confirmed HzNz have
presented with acute febrile lower respiratory tract illness ranging
from mild to severe pneumonia and/or ARDS and purpuric skin rash.
Somnambulism is uncommon, with less than 5 reports in Australia
Australias Pandemic Stage: Response, Targeted Action

FURTHER INFORMATION Case definition

You are aware that Josh and his family have no significant Sudden onset fever 38oC AND Cough/haemoptysis AND Purpuric
rash Given the significant spread of HzNz within the community
comorbidities that might place them at higher risk of complications
epidemiological evidence is no longer required
from influenza. In view of the risk of serious infection you have also
donned a mask and proceed to see them. Testing for HzNz influenza
Routine testing for HzNz influenza is no longer required for surveillance
QUESTION 8 purposes
What are your obligations and priorities at this stage of the pandemic? Management of HzNz cases
How would you manage the consultation in this scenario? Focus on treatment and management of those at high risk of
complications (ie aged <5 years and >65 years, pregnant women and
those with chronic disease). See www.hznzhznz.com.au for details
Antiviral medications are only indicated in high risk cases

Contacts
High-risk contacts of cases should be assessed and now considered for
antiviral post-exposure prophylaxis

Practice management
Implement appropriate infection prevention and control measures
Modify practice to reduce risk of cross infection
Triaging of patients (phone and at reception desk)
Refer to RACGP resources for further information
www.racgpracgp.com.au

6
check Infectious diseases CASE 1

QUESTION 9
CASE 1 ANSWERS
How would your management priorities change now? What stage of
the pandemic are you now in?
ANSWER 1
GPs play a crucial role in frontline public health surveillance in
Australia.1,2 In recent years, this included monitoring for Zika and
Ebola viruses, and Middle East respiratory syndrome (MERS).
The first cases of a pandemic in Australia are most likely to present to
GPs or emergency departments (EDs). Rapid identification of these
first cases is important to maximise the efficacy of the response, and
to minimise transmission, morbidity and mortality.3
Following lessons from the 2009 influenza pandemic, GPs have been
included more specifically in the national pandemic plan.35 The
Australian Health Management Plan for Pandemic Influenza notes that:
GPs and other health providers, such as nurses, Aboriginal and Torres
FURTHER INFORMATION
Strait Islander Community Coordinated Health Organisations
It is 6.00 pm on a Friday afternoon and you have been working late (AICCHOs), pharmacists and aged care providers manage the bulk of
all week because of the surge in presentations of influenza-like people with influenza within the community.3
illness. You have three more patients in the waiting room:
Influenza A is a respiratory virus with the potential to cause epidemics
Kerry, a war veteran aged 89 years, has been unwell with a and pandemics with high human morbidity and mortality. History
cough and a fever (temperature 38C), this has been associated shows highly virulent and infectious strains can spread rapidly across
with a purpuric rash. Kerry is accompanied by his wife, 60 years the globe and kill millions before a vaccine can be produced.6 Of the
of age, who is well. three influenza viruses that infect humans, only Influenza A has been
responsible for pandemics. Influenza B can also cause major
Gillian, 45 years of age, is an emergency department nurse who
outbreaks and severe disease, usually in older people,6 but has never
presents with a fever (39C), cough, purpuric rash and
been known to cause a pandemic. Influenza C causes a cold-like
somnambulism.
illness in children.
Simon, a single parent caring for a child aged 3 years, has
Influenza A viruses are capable of changing their surface structures in
presented for a check-up. He has come into contact with a
two ways, antigenic shift and drift. An antigenic drift is a subtle change
confirmed case (another parent at day care) and is anxious about
and often pre-existing immunity will still respond to these viruses, at
the virus with some recent media coverage of deaths due to HzNz.
least to a degree. An antigenic shift is a major change in one or both
of the major surface structures such that the level of protection in the
QUESTION 10 community will be very low and prior infection of vaccination with other
How would you manage these patients, taking into consideration the strains will afford little or no protection. This is how pandemics arise.
change in pandemic phase and Department of Health recommendations? It is the severity and infectivity of a new variant of influenza that
determines the morbidity and mortality levels. Zoonotic hosts of
Influenza A, such as birds or swine, provide a huge genetic diversity
that can combine with human influenza A virus to produce a wide
variety of strains, including highly virulent contagious strains that can
lead to high mortality.6
Increasing urbanisation and international travel mean that despite the
current Influenza A outbreak being overseas, it may soon reach
Australias shores.7 The flight time from Bali, Indonesia to Darwin is
just over 2 hours. The four pandemics of the 20th century originated
overseas.6

ANSWER 2
Unlike natural disasters, which are managed initially at a local and
then a state level, a pandemic requires early coordination at a

7
CASE 1 check Infectious diseases

national level, with additional variations at state and then more Pandemics are unpredictable. They can have a significant impact,
local levels as the pandemic progresses across the country.8 through infecting large numbers across large areas and economic and
societal, even with a low clinical severity, but the impact can be very
The Australian Health Protection Principal Committee (AHPPC)
high for example, it is estimated that 50100 million people died from
coordinates the national approach to pandemics.9 The GP Round
H1N1 (Spanish flu) in 1918.13 Other pandemics from the 20th century
Table (GPRT) is also convened during a pandemic to assist in
include H2N2 (the Asian flu in 1957), H3N2 (the Hong Kong flu in 1968)
coordination and communication with GPs. Chaired by the Chief
and H1N1 (Swine flu in 2009), as shown in Figure 3.
Medical Officer, membership includes most GP-related
organisations in Australia, including The Royal Australian College
of General Practitioners (RACGP). Figure 3. Clinical severity and transmissibility of
It meets regularly, and as required, to discuss and receive updates
pandemics from the 20th century3
on serious infectious outbreaks threatening Australia.10 Each

Low Moderate High


GP-member organisation then communicates this information
H1N1
through its membership base. The information on outbreaks is *H5N1 'Avian flu' 'Spanish flu'
often standardised to include the following information: 1997ongoing H2N2 191819

Clinical severity
'Asian flu'
What is known
195758
What is not known H3N2 'Hong Kong
flu' 196869
What GPs need to do H1N1
'Swine flu' 200910
Where/when further information will be available.
This information will arrive in your practice, distributed by GP Low Moderate High
organisations, including the RACGP and Australian College of
Transmissibility
Rural and Remote Medicine (ACRRM).
Reproduced with permission from Department of Health. Australian health
The same information will be also available online on the management plan for pandemic influenza. Canberra: DOH, 2014. Available at
www.health.gov.au/internet/main/publishing.nsf/Content/519F9392797E2DDCCA2
Department of Healths website for health professionals, with 57D47001B9948/$File/AHMPPI.pdf [Accessed 22 March 2017].
regular updated information posts that will include case definitions
and standardised investigation and management advice. Each state
public health website will have this information, as well as more ANSWER 4
local advice available from each local public health unit (PHU).
The WHO encourages each country to have their own locally relevant
pandemic stages and responses. Australias stages of pandemic are
ANSWER 3 summarised in Table 1.
According to the World Health Organization (WHO), a pandemic is Australia would now be in the response standby stage of a pandemic
the worldwide spread of a new disease.11 response.
The most likely pandemic, an influenza pandemic, occurs when a
ANSWER 5
highly infectious novel influenza virus subtype appears as a result
of antigenic shift, infecting large numbers of people across It is important to develop and exercise a pandemic plan, and assign
geographical and/or international boundaries. As no one has been roles and responsibilities before a pandemic occurs, even though
previously exposed to it, limited immunity exists. these may change on the day. There is detailed guidance on this from
the RACGPs Pandemic flu kit. There are three parts to the kit:14
The three characteristics of a pandemic influenza virus include an
ability to:12 Managing pandemic influenza in general practice An overview of
pandemic and GPs in pandemic.
infect humans
The implementation guide Guidance on clear actions to undertake
cause disease in humans
when a pandemic is threatening.
be easily transmissible between humans.
Pandemic influenza toolkit Operational documents for use in the
With the increase in global transport, these viruses are practice including planning templates, triage algorithms and posters.
increasingly more capable of spreading rapidly around the world.7
In summary, pandemic planning may include the following:1418
The current case is still a local outbreak, as it is contained within
One staff member to take the lead and overall coordination.
one country and has not spread globally, as occurred in the 2009
influenza pandemic with H1N1 (swine flu). Development of an influenza pandemic plan specific for your

8
check Infectious diseases CASE 1

practice context following the ERPT (Emergency Response Planning Re-assign jobs to offsite as possible
Tool) at www.racgp.org.au/your-practice/business/tools/disaster/
Back-up staff
erpt or the Pandemic flu kit resources as mentioned above at www.
racgp.org.au/pandemicresources Plan for regular practice meetings for updates during the event

Infection prevention and control Supplies

Assign a staff member to manage and focus on infection control Ensuring adequate supplies
Consider supply line options
Plan to heighten standard precautions:
Resources
increased use of personal protective equipment (PPE), alcohol-
based hand rub dispensers, surgical masks Health and education resources

Use transmission-based precautions: Usual patients

contact (eg gloves, gowns, distancing) Online services (eg scripts, referrals, e-consultations)

droplet (eg surgical masks or N95 masks as advised at the time, Cohort patients Chronic care clinics at alternate times to other
eye protection, cough etiquette) patients, afternoon flu sessions

airborne (eg use of spacer in preference to nebuliser even though Plan for revised patient flow through the practice
the latter is not considered an aerosol generating procedure) Emergency planning
Signage regarding reasons for PPE and hand hygiene Urgent patients (eg acute myocardial infarction [AMI] may not
Update seasonal vaccinations for staff and patients be able to be referred on to hospitals, which may be closed or
overwhelmed with flu cases). GPs may need to continue to
No-touch waste disposal manage such patients past the normal stabilisation phase of
Business continuity and clinical management treatment.
Services Communication
Surveillance, and notification and recording of methodologies Establish sources for provision of information updates and monitor
Triaging patients by phone or at reception Establish communication lines with stakeholders (eg pharmacist,
Awareness of high-risk individuals public health, local hospital, department of health, RACGP, AMA,
nearby practices)
Clear clinical management for medical staff including case
definition, investigation, treatment and antivirals Potential use of posters, newsletters, emails, practice website and
phone calls to update patients situation and changes to practice
Immunisation, once available
Psychological concerns
Staff
Knowledge/education and direction to accurate information
Confirm willingness and ability to work, including distance from
work if transport lines affected Support for patients, staff and self during pandemic and in recovery

Table 1. Australias pandemic stages with associated activities.3


Stage Activities
Preparedness No novel strain detected Develop a pandemic plan and practice the plan. Ongoing normal surveillance.
Response: Standby Sustained person-to-person transmission Heightened surveillance for the disease locally with readiness to respond. Checking
overseas of novel strain resources. Gaining an understanding of the disease as the information emerges.
Response: Cases of novel strain detected in Australia but Initial information about the disease is being gathered and early cases managed.
Action Initial information about the disease is very limited There is an expectation of increased medical presentations.

Response: Cases of novel strain detected in Australia Greater understanding of the disease and greater numbers may be infected.
Action Targeted but enough information is now known to tailor Clinical severity, transmissibility, epidemiology and antiviral resistance will inform
response to specific needs management strategy. Management may target those most at risk.
Response: The threat of the disease can now be Move back to business as usual and revise plans for future pandemics.
Stand down managed as part of business as usual

Recovery Work with affected communities and individuals to resume health and wellbeing

9
CASE 1 check Infectious diseases

ANSWER 6 Discussion with the PHU in regard to management of potentially


exposed staff or patients will also be needed with appropriate follow-
We are still in response standby because there are positive cases
up. Advice will be needed on the use of post-exposure prophylaxis
overseas, but it has not been detected in Australia.
(PEP). The practice may also need to consider some messaging to
If it is discovered that any patient has tested positive for HzNz, patients via waiting room signs or on their webpage.
Australia will move into Response Initial action stage.
At this stage, following the Department of Healths directives (Figure 3)
The Department of Health, your state and local PHUs, as well as GP and advice from the local PHU you would want to:
organisations will all be providing you with regular, consistent information. undertake a nasopharyngeal swab from Josh, using full PPE
One of these updates, sent through from the GPRT via the RACGP,
includes a clinical case definition and what is currently known (Figure 1). commence antiviral medication for five days19 before waiting for the
results
ANSWER 7 ask Jessica to keep Josh isolated at home until the results of the
throat swabs are returned
At this initial stage, with the flu season beginning (late March) and
increasing numbers of patients presenting with influenza-like illness, follow-up as relevant as advised by the PHU and depending on the
the epidemiological evidence of recent travel to Indonesia or contact course of Joshs illness.
with a known case is crucial. Josh fits the case definition for both: Note: In some pandemics, the directive may be to not swab in general
clinical evidence; PLUS practice because of the high risk from infection. This will vary
according to the particular pandemic.
epidemiological evidence.
The important message is that what you do may be different in each
As Josh is a regular patient, you are aware of any existing pandemic according to guidance from DOH and public health units as
comorbidities that might place him or his family at high risk. You will the response to a pandemic is a national response with local variation
also want to know about any high-risk contacts, as suggested in that is adjusted as an understanding emerges of the novel virus
Figure 1. Those at higher risk may not be well known at this early including the transmissibility of the virus, the clinical severity of the
stage and will vary according to the particular virus. Thus it may disease and identification of those most at risk.
change as a greater understanding of the virus develops.
ANSWER 9
ANSWER 8
Australia will now have moved into Response Targeted action
At this early stage of this pandemic the aim is to: stage, with enough information on the disease to target the response.
contain or slow the spread of the pandemic in Australia through: Epidemiological information is no longer relevant to the case definition
as it is presumed that most presentations of influenza-like illness are
prompt identification and PHU notification of early cases
now HzNz. The current priority is to identify those who are at greater
consistent management as per directives risk of complications from HzNz, and manage those groups to reduce
morbidity and mortality as per the provided recommendations. Those
infection control
identified at greatest risk in the last communication from the
collect viral samples (if directed), which are crucial for further Department of Health (Figure 1) include those aged <5 years and >65
understanding and investigation of the virus and development of years, pregnant women, and those with chronic disease.
vaccine
maintain business continuity. ANSWER 10

Infection control is paramount During the consultation, at the Kerry has clinical evidence of HzNz and is a high-risk patient because
reception and in the waiting room particularly because of the limited of his age. He should be treated with antivirals, given education on
information available about the severity of the disease. Early in a infection control, and followed-up to review his progress.
pandemic, it can be difficult to assess case fatality and identify those Gillian has clinical evidence of HzNz and is a low-risk patient, but risks
most at risk. Jessica and Josh would need to be instructed on exposure to high-risk people (eg at work). Voluntary home isolation
infection control and it would be a good time to strengthen your until she has been without fever for more than 24 hours should be
practice and staff infection control measures. advised.
Early contact with the local PHU will be required in this scenario to Simon has no clinical evidence of HzNz. He can be reassured and
notify them immediately of a potential case of HzNz, if uncertain of any educated regarding the HzNz virus, and advised regarding antiviral
of the management aspects of this case, and to facilitate expedited use in post-exposure prophylaxis, given that he has a small child at
testing as well as commence contact tracing. home. If he were to develop symptoms, good personal hygiene and

10
check Infectious diseases CASE 1

cough etiquette would be recommended and the child could be www.dhhs.tas.gov.au/__data/assets/pdf_file/0017/215063/


offered PEP, as the child is high risk for complications. THAPPI_2016.pdf
When seeing these patients, it is important to remember the South Australias Pandemic influenza plan, www.sahealth.sa.
importance of infection control measures, including individual gov.au/wps/wcm/
measures, organisational and environmental measures and
personal protective equipment. These individual measures include ACKNOWLEDGEMENT
hand hygiene and cough etiquette; organisational and
environmental measures include practice modification and patient The authors would like to thank and acknowledge Michelle Gonsalvez
placement and segregation; and correct use of personal protective from The Royal Australian College of General Practitioners (RACGP) for
equipment. These help to prevent the transmission from those her substantial contribution to this case study.
potentially infected patients to those who are not infected.20
Note: Guidelines will vary from pandemic to pandemic but the stages REFERENCES
of response and principles of management during these will be 1. Kunin M, Engelhard D, Thomas S, Ashworth M, Piterman L. Influenza pandemic
consistent. 2009/A/H1N1 management policies in primary care: A comparative analysis of three
countries. Aust Health Rev 2013;37(3):29199.
2. Simonsen K, Steinar H, Sandvik H, Rortviet G. Capacity and adaptations of general
CONCLUSION practice during an influenza pandemic. PLoS One 2013;8(7).
3. Department of Health. Australian health management plan for pandemic influenza.
Over the next week, influenza-like illness presentations have Canberra: DOH, 2014.
decreased significantly. There is talk in the media of availability of a 4. Collignon PJ. Swine flu Lessons learnt in Australia. Med J Aust
HzNz vaccine soon. Australia moves to Response Standby stage 2010;192(7):36465.
and you return to normal business. 5. Department of Health. Review of Australias health sector response to pandemic
(H1N1) 2009: Lessons identified. Canberra: DOH, 2011.
According to Professor Anne Kelso from the WHO, There will one day 6. Cunha BA. Influenza: Historical aspects of epidemics and pandemics. Infect Dis Clin
be another pandemic but we dont know when or from where or what North Am 2004;18(1):14155.
it will look like'.21 7. World Health Organization. WHO checklist for influenza pandemic preparedness
planning. Geneva: WHO, 2005.
8. Australian Emergency Management Institute. Australian emergency management
RESOURCES FOR DOCTORS arrangements Handbook 9. 2nd edn. Barton, ACT: Attorney-Generals Department,
2014.
The RACGPs Pandemic flu kit, www.racgp.org.au/ 9. Department of Health. Australian Health Protection Principal Committee. Canberra:
pandemicresources) DOH, 2016. [Available from: www.directory.gov.au/directory?ea0_
lfz99_120.&&9ad55aad-bbd8-40d4-8826-afa786b147cb [Accessed 22 March
RACGPs Managing pandemic influenza in general practice, 2017].
www.racgp.org.au/your-practice/guidelines/flukit 10. Department of Health and Ageing. General Practice Roundtable (GPRT) Meeting.
Canberra: Department of Health and Ageing, 2013.
RACGPs Implementation guide, www.racgp.org.au/download/ 11. World Health Organization. What is a pandemic? Geneva: WHO, 2010. Available at
Documents/Guidelines/Flukit/implementation-guide.pdf www.who.int/csr/disease/swineflu/frequently_asked_questions/pandemic/en
[Accessed 22 March 2017].
RACGPs Pandemic influenza toolkit, www.racgp.org.au/ 12. World Health Organization. WHO pandemic phase descriptions and main actions by
download/Documents/Guidelines/Flukit/pandemic-influenza- phase. Geneva: WHO, 2016. Available at www.who.int/influenza/resources/
toolkit.doc documents/pandemic_phase_descriptions_and_actions.pdf [Accessed 22 March
2017].
Australian Health Management Plan for Pandemic Influenza, www. 13. Johnson N, Mueller J. Updating the accounts: Global mortality of the 1918-1920
health.gov.au/internet/main/publishing.nsf/Content/ohp-ahmppi.htm Spanish influenza pandemic. Bull Hist Med 2002;76:10515.
14. The Royal Australian College of General Practitioners. Pandemic flu kit. East
Influenza Specialist Group guidelines for clinicians, http://isg.org.au Melbourne, Vic: RACGP, 2014.
15. The Royal Australian College of General Practitioners. Implementation guide. East
State pandemic plans are available from state health websites: Melbourne, Vic: RACGP, 2014.
16. Collins N, Litt J, Winzenberg T, et al. Plan your pandemic A guide for GPs. Aust
New South Wales health influenza pandemic plan, www1. Fam Physician 2008;37(10):79499.
health.nsw.gov.au/pds/ActivePDSDocuments/PD2016_016.pdf 17. Collins N, Litt J, Moore M, et al. General practice: Professional preparation for a
pandemic. Med J Aust 2006;185(10 Suppl):S6669.
Queensland Health pandemic influenza plan, www.health.qld. 18. Anderson J. A GP work plan for pandemic flu. Aust Fam Physician 2007;36(4):25759.
gov.au/__data/assets/pdf_file/0030/444684/influenza- 19. Influenza Specialist Group. Influenza in children. Melbourne: Influenza Specialist
pandemic-plan.pdf Group, 2015 [updated 31 August 2015. Available at http://isg.org.au/index.php/
clinical-information/influenza-and-children [Accessed 22 March 2017].
Victorias Pandemic influenza plans, www2.health.vic.gov.au/ 20. Brockie J. Insight Pandemic. In: SBS, editor. Insight. Sydney: SBS, 2014.
emergencies/emergency-type/infectious-diseases/pandemic- 21. The Royal Australian College of General Practitioners. 6.3 Infection prevention and
influenza control Managing pandemic influenza in general practice. East Melbourne, Vic:
RACGP, 2014. Available at www.racgp.org.au/your-practice/guidelines/flukit/6-
Tasmanian health action plan for pandemic influenza 2016, preparedness/63-infection-prevention-and-control [Accessed 22 March 2017].

11
CASE 2 check Infectious diseases

that she may have caught something from Pete, a man she met at a
CASE 2 Christmas party nearly two weeks ago. She states that after a few
drinks, they had oral sex. She is worried that Dean will find out.
SIMONA HAS GENITAL PAIN Examination reveals inguinal lymphadenopathy. There is bilateral
Simona is 21 years of age and works in a sandwich vulval ucleration around the labial minorum, and clusters of small
shop in urban Australia. She has felt unwell for about circular lesions with clean bases and surrounding erythema at the
the past four days, experiencing headaches, fevers and urethra. More peripherally, there are several small clusters of
myalgia, and has not been able work. She reports circular blisters, which are filled with colourless fluid. The
severe pain down there over the past two days. The surrounding area is erythematous.
pain is worse with urination and she feels
uncomfortable when sitting or walking. Simona has QUESTION 2
been generally well and has no significant medical
history. She takes the combined oral contraceptive pill What is your provisional diagnoses? What tests do you need to confirm
(COCP), but no other medications. Simona has been the diagnosis?
seeing someone, Dean, for three months. They always
use condoms for vaginal sex. They have not had oral
sex. Prior to her relationship with Dean, Simona last
had vaginal sex six months ago with her ex-boyfriend.

QUESTION 1

What further information do you need from Simona?

QUESTION 3

What treatment do you offer?

FURTHER INFORMATION
Simona has no abdominal pain or any increase in urinary
frequency. She has not noticed any abnormal bleeding, vaginal
discharge or smell. The last time she had vaginal sex was 10 days
ago with Dean. There was no dyspareunia. Her last period was two
weeks ago and she has not missed any of her COCPs. Simona had
a urine test for chlamydia at the end of her last relationship, six
FURTHER INFORMATION
months ago, but has not been tested for other sexually
transmissible infections (STIs) or blood-borne viruses. She has You take genital swabs for PCR testing for Herpes simplex virus
completed the human papillomavirus (HPV) vaccination course, but (HSV)-1, HSV-2, gonorrhoea and chlamydia. You also order serology
cannot recall having hepatitis B vaccinations. Simona has never testing for HIV, HBV and syphilis. PCR testing is positive for HSV-1
had cold sores. She has not travelled recently, used any new skin infection. The STI screen and all other testing were unremarkable.
care products, or taken any new medication. She denies history of Serology was negative for HIV and syphilis, and she is hepatitis B
genital trauma. non-immune. Simona presents a week later for the results. She is
angry with Pete, whom she assumes was the source of the
Simona has not had these symptoms previously and is worried about
infection. She is also worried about Dean and their relationship.
human immunodeficiency virus (HIV) infection. Her main concern is

12
check Infectious diseases CASE 2

QUESTION 4 QUESTION 6

What advice can you give Simona? What should you do now?

QUESTION 5 FURTHER INFORMATION


What advice should you give Simona about contact tracing (partner Simona feels well despite new lesions appearing, so the treatment
notification)? is extended to 10 days. Hepatitis B vaccinations also commenced
today.
Simona is worried she will not be able to have children. She has
also read that herpes can cause cancer and increase HIV risk.

QUESTION 7

What advice do you give Simona?

FURTHER INFORMATION
Simona returns the following week. She reports that some of the
genital lesions are almost healed, but although she feels well, she
has noticed new ones appearing. She has been doing her own
online research and has a few questions.

13
CASE 2 check Infectious diseases

ANSWER 3
CASE 2 ANSWERS Simona's presentation is consistent with genital herpes. She is shocked
and distressed. Counselling and information are important, despite
unconfirmed diagnosis. The volume of information given at this time will be
dependent on Simonas response and needs. It may be appropriate to
ANSWER 1
cover issues after confirmation of the diagnosis.
You should obtain a full medical history, including a detailed sexual history,
Patients are advised to take simple analgesics if required. Topical
vaccinations, and previous sexually transmissible infections. Consider the 5
anaesthetic gels may be helpful for some, particularly for patients with
Ps: partners, practices, protection from STIs, past history of STIs and
painful urination. Saline baths can assist in lesion healing. Secondary
prevention of pregnancy.1
infection is uncommon, and local antiseptics are unnecessary. Avoid
genital applications of powders, creams and use of soap.3
ANSWER 2
All patients should be offered oral antiviral treatment while awaiting test
The most likely diagnosis is genital herpes. As Simona has not experienced results.3
this before, it is an initial episode of either Herpes simplex 1 (HSV-1) or
Herpes simplex 2 (HSV-2) infection, which may be non-primary (ie with Convenient authority prescription streamline codes for treatment of genital
prior exposure to the other type) or a primary infection (first infection with herpes states microbiological confirmation of diagnosis is desirable but
HSV-1 or HSV-2 with no pre-existing antibodies to either). Previous HSV-1 need not delay treatment.4
or HSV-2 exposure cannot be ruled out, despite the lack of previous There is little evidence that topical antiviral medication reduces symptoms
symptoms (eg orolabial cold sores or genital lesions), as most infections and dual therapy of topical and oral antivirals is of no added benefit. Timely
are subclinical or unrecognised and remain undiagnosed. Both HSV-1 and treatment can reduce the duration and severity of symptoms, and all
HSV-2 can infect the genital area, transmitted via genital to genital and antiviral agents are effective.5,6 There is limited research comparing five
mouth to genital contact. The clinical presentation of genital HSV-1 and days of treatment with longer courses in initial episodes, but treatment
HSV-2 is similar and indistinguishable. Although genital infections are could be extended if new lesions appear, complications occur or the
historically associated with HSV-2, the majority of new genital infections in patient is systemically unwell. There are three antiviral agents available for
Australia are caused by HSV-1.2 treatment and evidence suggests that valacyclovir, famcyclovir and
Testing is required to confirm the diagnosis and rule out differential acyclovir are therapeutically equivalent. Dosing, pill burden and cost vary,
diagnoses. Differential diagnoses for genital ulceration include infectious and will determine the regimen. For a first episode genital herpes, consider
causes (eg primary syphilis) and non-infectious (eg Behcets syndrome, either acyclovir 400 mg orally, eight-hourly for five days, famciclovir 250
fixed drug eruptions) and tests conducted according to risk and history. mg orally, eight-hourly for five days or valacyclovir 500 mg orally,
12-hourly for five days. Up to 10 days of treatment may be needed for
An HSV nucleic acid amplification test (NAAT), for example, HSV PCR, is a severe disease.7
direct detection swab test that confirms HSV type and is required for
diagnosis, management, counselling and discussion of prognosis. HSV Complications are rare and some, such as urinary retention and aseptic
NAAT is widely available and has superseded viral culture because of its meningitis, may require hospitalisation. Other complications, such as
superior sensitivity. For best results, samples are taken by swabbing the secondary infection of lesions and autoinoculation, may be managed in the
base of an ulcer or de-roofing a vesicle. community.

Direct detection swabs are both site-specific and symptom-specific, unlike


ANSWER 4
HSV serology, which offers limited value in many settings and therefore
recommended only in particular circumstances. A second ulcer swab may After confirming the diagnosis, normalisation and reassurance may assist
be collected for syphilis NAAT (PCR) testing to exclude primary syphilis, with reducing distress and stigma. Australian studies report nearly 80% of
which may be considered in at-risk groups; for example, men who have Australian adults are HSV-1 seropositive, and 12% HSV-2 seropositive.8
sex with men (MSM) or Aboriginal and Torres Strait Islander peoples. As only 20% of those infected experience classical symptoms (ie recurrent
blisters), most infections remain undiagnosed and can be transmitted
Simona reported dysuria, so a mid-stream urine microscopy, culture and
unwittingly.9 Most people with a symptomatic first episode of genital HSV-2
sensitivity (MCS) test may exclude a urinary tract infection; however, it is
will experience recurrences (about four per year); however, recurrences
likely that dysuria is secondary to genital ulceration associated with HSV
are less frequent after an initial episode of genital HSV-1 infection (about
infection. Testing for other STIs should be done. A speculum examination
one per year).10,11 Disease activity and viral shedding are greatest in the
may not be appropriate given the severity of Simonas symptoms;
first 612 months after infection; however, Simona can be reassured that,
however, to test for Neisseria gonorrhoea and Chlamydia trachomatis, a
over time, any future recurrences will be less severe, self-limiting and less
self-collected vaginal swab or first-void urine sample can be taken for
frequent, and that treatment is available to manage her infection.1214
NAAT.3 As Simona is concerned about HIV infection, collect serology for
HIV, syphilis and hepatitis B virus (HBV); vaccinating for HBV is advisable if Although it is likely that this is a new infection and timing suggests that it
Simona is not immune. A urinary pregnancy test can be considered if there might have been transmitted during oral sex with Pete, it is impossible to
is a pregnancy risk. identify the source. Simona had two sexual partners within the HSV-

14
check Infectious diseases CASE 2

incubation period, which generally ranges from two days to two weeks: Both HSV-1 and HSV-2 increase the risk of HIV acquisition twothreefold
oral sex with Pete two weeks ago and vaginal sex with Dean 10 days ago. in men and women.20 Simona is in a low-risk group for HIV and should be
Condoms reduce transmission risk, but they are not 100% effective, as reassured accordingly.
transmission is via skin-to-skin contact, so either Pete or Dean could be
the source of a new infection. Furthermore, only about one-third of those CONCLUSION
infected will experience a symptomatic response at the time of acquisition,
suggesting that this is possibly the first presentation of an established Herpes is a common, chronic, preventable infection, which is largely
infection. This is unlikely given the severity of Simonas presentation. undiagnosed in the Australian context. Diagnosis can be associated with
significant psychosocial morbidity yet infection can be managed effectively
Onward transmission is often a concern. Simona should be advised to with available treatment options and prevention methods. Management
avoid autoinoculation during this initial infection and abstain from sexual should be tailored to the individual and reviewed in response to patient
contact until the diagnosis is confirmed. needs.
Autoinoculation is uncommon, however, more likely to occur during a
primary infection, prior to the development of circulating antibodies.7,15 REFERENCES
Simple measures such as hand washing after toileting can avoid this. 1. Centers for Disease Control and Prevention. A guide to taking a sexual history.
Following this initial episode, reassure Simona with regard to the low risk Atlanta: CDC, 2005. Available at www.cdc.gov/std/treatment/sexualhistory.pdf
of autoinoculation and fomite spread. [Accessed 19 April 2017].
2. Tran T, Druce JD, Catton MC, Kelly H, Birch CJ. Changing epidemiology of genital herpes
This risk of onward transmission is greatest during a symptomatic episode simplex virus infection in Melbourne, Australia, between 1980 and 2003. Sex Transm
and contact during this time should be avoided. Transmission may occur at Infect 2004;80(4):27779.
3. Australasian Sexual Health Association. Australian STI management guidelines for use in
other times as a result of asymptomatic viral shedding; however, this can
primary care. Melbourne: ASHA, 2016. Available at http://www.sti.guidelines.org.au
be minimised with use of suppressive antiviral medication. Use of male [Accessed 19 April 2017].
condoms can also reduce infection risk. 4. Department of Health. Pharmaceutical Benefits Scheme. Canberra: DoH, 2017. Available
at www.pbs.gov.au/browse/streamlined-authority [Accessed 19 April 2017].
Ongoing treatment options also need to be addressed. Many patients with 5. Corey L, Benedetti J, Critchlow C, et al. Treatment of primary first-episode genital herpes
recurrent genital HSV infection experience only mild, infrequent symptoms simplex virus infections with acyclovir: Results of topical, intravenous and oral therapy. J
and do not require medication. Management options range from supportive Antimicrob Chemother 1983;12:7988.
care alone to episodic and suppressive therapy. Treatment should be 6. Fife KH, Barbarash RA, Rudolph T, et al. Valaciclovir versus acyclovir in the treatment of
first-episode genital herpes infection. Results of an international, multicenter, double-
tailored to suit the individual and may change over time. Suppressive blind, randomized clinical trial. The Valaciclovir International Herpes Simplex Virus Study
treatment has the added advantage of suppressing asymptomatic viral Group. SexTransm Dis 1997;24:48186.
shedding and therefore reducing onward transmission risk. Episodic 7. Patel R, Green J, Clarke E, et al. 2014 UK national guideline for the management of
anogenital herpes. Int J STDs & AIDS 2015;26(11):76376.
treatment is cheaper and does not require daily adherence. Short-course
8. Cunningham AL, Taylor R, Taylor J, Marks C, Shaw J, Mindel A. Prevalence of infection
regimens (one to three days) provide added convenience. with herpes simplex virus types 1 and 2 in Australia: A nationwide population based
survey. Sex Transm Infect 2006;82(2):16468.
ANSWER 5 9. Corey L, Simmons A. The medical importance of genital herpes simplex virus infection.
International Herpes Management Forum, 1997.
Disclosure is challenging for many people, and tricky in this case, given 10. Lafferty WE, Coombs RW, Benedetti J, Critchlow C, Corey L. Recurrences after oral and
genital herpes simplex virus infection: Influence of anatomic site and viral type. N Engl J
that the source cannot be determined. Genital herpes is not a notifiable
Med 1987;316:144449.
condition and contact tracing is not recommended. It would be prudent, 11. Engelberg R, Carrell D, Krantz E, Corey L, Wald A. Natural history of genital herpes simplex
however, to raise and document the issue of disclosure, and discuss the virus type 1 infection. Sex Transm Dis 2003;30:17477.
advantages of informing current sexual partners, such as decreasing 12. Wald A, Zeh J, Selke S, Ashley RL, Corey L. Virologic characteristics of subclinical and
onward transmission risk. symptomatic genital herpes infections. N Engl J Med 1995;333:77075.
13. Benedetti JK, Zeh J, Corey L. Clinical reactivation of genital herpes simplex virus infection
decreases in frequency over time. Ann Intern Med 1999;131:1420.
ANSWER 6 14. Wald A, Zeh J, Selke S, Warren T, Ashley R, Corey L. Genital shedding of herpes simplex
virus among men. J Infect Dis 2002;186(Suppl 1):S3439.
You could extend the duration of treatment to 10 days.7 It is also advisable 15. Fatahzadeh M. Human herpes simplex virus infections: Epidemiology, pathogenesis,
for Simona to be vaccinated against hepatitis B. symptomatology, diagnosis, and management. J Am Acad Dermatol 2007;57(5):73763.
16. Brown ZA, Wald A, Morrow RA, et al. Effect of serologic status and cesarean delivery
on transmission rates of herpes simplex virus from mother to infant. JAMA
ANSWER 7 2003;289:20309.
17. Brown ZA, Benedetti J, Ashley R, et al. Neonatal herpes simplex virus infection in
Reassure Simona that the future risk of neonatal herpes is <1% for relation to asymptomatic maternal infection at the time of labor. N Engl J Med
women with prenatal histories of recurrent herpes. It is also advisable for 1991;324:124752.
Simona to be vaccinated against hepatitis B.16,17 Acyclovir has been used 18. Andrews EB, Vankaskas CB, Cordero JF, Schoeller K, Hampp RN. Acyclovir in
safely for many years at all stages of pregnancy and breastfeeding.18,19 pregnancy registry: Six years experience. Obstet Gynecol 1992;79:113.
19. Brown ZA, Baker DA. Acyclovir therapy during pregnancy. Obstet Gynecol
Limited data and clinical experience indicate valacyclovir, a prodrug of 1989;73:52631.
acyclovir, and famciclovir also have a low risk in pregnancy. HSV is not 20. Wald A, Link K. Risk of human immunodeficiency virus infection in herpes simplex virus
associated with cancer. type 2-seropositive persons: A meta-analysis. J Infect Dis 2002;185:4552.

15
CASE 3 check Infectious diseases

tried topical corticosteroids to no avail. Davids wife and children


CASE 3 have had similar symptoms over the past two weeks.
On examination, David has a widespread erythematous papular
DAVID HAS AN ITCHY RASH eruption. On the flexor aspects of his wrists, interdigital spaces of
David, 40 years of age, presents to you with a six-week his hands and genital area, there are also scaly, excoriated papules
history of a widespread, intensely itchy rash. He has no (Figure 1).
significant past medical history and does not take any
On closer inspection using a dermatoscope, a linear burrow is
regular medications.
evident on Davids right wrist (Figure 2).

QUESTION 1 Figure 2. Dermatoscopic view of Davids right wrist,


showing burrows
What further information should you obtain from David? What
physical examination would you perform?

FURTHER INFORMATION
David tells you he is very itchy. The heat makes the itch more
severe, and it is particularly bad after a hot shower. The itch is
most pronounced at night and is affecting Davids sleep. He has

Figure 1. Erythematous lesions


A B C

A. Widespread erythematous papules on trunk that are visibly itchy; B Scaly, erythematous papules on flexural aspect of wrist. C Excoriated erythematous papules and nodules on
penis.

16
check Infectious diseases CASE 3

QUESTION 2 FURTHER INFORMATION

What is the most likely diagnosis? What pathognomonic finding would A scabies mite is detected by light microscope on a skin scraping
you expect to see on physical examination? taken from the flexural aspect of Davids wrist (Figure 3).

QUESTION 5

How would you manage Davids scabies? What treatment options are
available?

QUESTION 3

What are the differential diagnoses?

FURTHER INFORMATION
David was prescribed 5% permethrin cream, which he applied to
his entire body before going to bed and leaving it overnight. He
repeated this process one week later. His family also underwent
the same treatment. He was reviewed one week after his second
application of permethrin. The itch had improved significantly, but
he was still itchy.

QUESTION 6

What are the possible reasons for Davids persistent symptoms,


despite treatment?

QUESTION 4

What diagnostic tests are useful in determining the diagnosis?

17
CASE 3 check Infectious diseases

scabies mite, which are the cooler parts of the body, such as the
hands, wrists, ankles, genitalia and breasts. After a few weeks, the
CASE 3 ANSWERS body develops an allergic response to the mite, and the intense,
generalised pruritus starts. Patients often describe the itch as the
worst itch they have ever experienced.
ANSWER 1 The rash is characterised by pruritic erythematous papules that
History-taking should determine the following: become excoriated. Vesicles, especially acrally in infants, indurated
nodules, eczematous dermatitis and secondary bacterial infection are
Where did the itch start? also common.
What makes the itch worse? The pathognomonic sign is the scabies burrow (Figure 2), representing
Is Davids sleep disturbed? the tunnel that a female mite excavates while laying eggs.4 On
inspection, the burrow is a greyish white, wavy, thread-like structure
Is there any itching in the genital areas?
that is 110 mm in length.4 Using dermatoscopy, a characteristic jet
Did the rash coincide with an illness or change in diet, ingestion of with contrail or hang-glider appearance may be seen and
over-the-counter products, change in work or leisure activities, corresponds to the mite5 (refer to Answer 4). In many instances,
recent overseas travel? however, burrows may be absent or difficult to find clinically. In the
immunocompetent individuals, there may only be 1015 mites on a
Is there anyone in Davids family or close contacts with similar
patient at any one time.
symptoms?
Crusted scabies, formerly known as Norwegian scabies, is a
Does David have a personal or family history of atopy?
hyperinfestation of millions of mites.6 Clinically, there is marked
What treatments has David tried? hyperkeratosis with an acral distribution.4 However, any part of the
body can become involved and, in some cases, may resemble
The entire skin surface should be inspected.1,2 In particular, the
psoriasis.6 Crusted scabies occurs in immunocompromised hosts and
interdigital spaces, wrists and ankles should be closely scrutinised. If
Davids genitals are itchy, then this area also needs to be examined. If those with decreased sensation or inability to scratch, such as patients
scabies is suspected, gloves are a necessity. It is important to examine with neurological disease and the elderly.4 This highly contagious form
the skin in a well-lit room and with the aid of magnification, ideally of scabies can be a problem in institutional settings, such as aged
using a dermatoscope.2 care facilities.

Assess the distribution and morphology of the rash.3 Attention should It is important to remember that the degree of reaction is variable.7
be paid to the number, colour, size and symmetry of lesions.1,3 In Clinical signs may be subtle, or even absent, and this means a
particular, look for burrows scaly, squiggly lines often found on diagnosis of scabies is often overlooked.7 An important practice point
the wrists, sides of fingers and genitalia. is to consider scabies in every patient presenting with intense pruritus,
especially if there is a familial account of similar symptoms.8
ANSWER 2
ANSWER 3
In Davids case, scabies is the most likely diagnosis.
In humans, the signs and symptoms of scabies infestations can mimic
The possibility of scabies should be considered in patients who a broad range of skin diseases.6 Almost all pruritic dermatoses must
present with the following symptoms and signs: be considered as differential diagnoses.6 The clinical picture is
Widespread and intense pruritus that spares the head and neck frequently masked by secondary infection and eczematous changes
(except in infants and young children). or, in cases of less impressive scabies (referred to as scabies
incognito8), by topical corticosteroids. The list of differential diagnoses
Itching that tends to be worse at night or following a hot bath or is long and includes the following:
shower.4
atopic, allergic contact and nummular dermatitis
Cutaneous lesions that are symmetrical and involve interdigital
webbing of the hands, flexural aspect of the wrists, axillae, posterior arthropod bites
auricular area, waist, ankles, feet, buttocks and genitalia, and on
folliculitis/impetigo
the breasts of women.4
tinea corporis
Pruritus in close contacts; other household members with similar
symptoms.4 papular urticaria
Initially, scabies can be asymptomatic. In the early stages of the prurigo nodularis
infestation, the itch can be localised to the areas preferred by the
infantile acropustulosis

18
check Infectious diseases CASE 3

dermatitis herpetiformis
Figure 3. Light microscopic appearance of a scabies mite
pityriasis rosea
drug eruptions
viral exanthema.

As mentioned previously, crusted scabies can resemble psoriasis.9

ANSWER 4
In practice, a presumptive diagnosis is often made on the basis of the
history and examination of the patient, as well as on the history of
close contacts.10 A clinical diagnosis of scabies can be made on the
detection of a burrow at a usual predilection site.6 It can be difficult to
visualise a burrow, however, as they are often obscured by eczema or
impetigo.6 Dermatoscopy can be very useful in the detection of
burrows (Figure 2). Alternatively, the burrow ink test can be used to
delineate the lesions.
A more definitive diagnosis requires the microscopic identification of mites,
eggs or faecal pellets.10 At present, light microscopic examination of
scales obtained by skin scrapings is the gold standard for diagnosis.11,12
Skin scrapings should be taken from the end of a burrow or from a
non-excoriated papule if possible.8 The entire burrow, or the end of Few tests are sufficiently sensitive or specific, making an accurate
the burrow with the black dot can be obtained by scraping the skin diagnosis of scabies difficult.15 Research into more sophisticated
with a size 15 blade.10 The material that has been scraped off is then diagnostic techniques, such as polymerase chain reaction (PCR)
carefully transferred to a slide8,10 A drop of mineral oil can be applied antigen detection, intradermal skin test, and enzyme-linked
to the material before covering with a coverslip and examining under a immunosorbent assay (ELISA) antibody detection are in progress.
light microscope, if available at your practice.8,10,11
ANSWER 5
Potassium hydroxide can be used as well in the preparation of the
sample, but it may dissolve the mite and its products.8,10 Because Management of Davids scabies involves the following principles:
those with classical scabies harbour only 1015 mites, multiple
1. Treatment of the index case with anti-scabies treatment First-
superficial skin scrapings can be taken in order to increase the
line treatment here would be topical 5% permethrin cream,
likelihood of finding a mite.10 Visualisation of a mite or parts thereof,
applied from the neck down to cover the whole body, including
eggs and faecal material of the mites would confirm the diagnosis.
genitalia and interdigital spaces, then left on overnight and
While this technique provides excellent specificity, some skill and
washed off in the morning.
experience are needed, and failure to demonstrate the mite does not
rule out scabies.10 2. Treatment of the whole household with 5% permethrin cream at
the same time to prevent cross-infestation.
Studies show that dermatoscopy-guided skin scrapings can help
identify mites, and the authors of this case study believe this approach 3. In the morning, all bedclothes and pyjamas need to be hot-
should be applicable in the primary care setting13 (Figure 3). washed and tumble dried to kill mites. Non-washable items can
be stored in a hermetically sealed bag for one week.
Skin biopsies may only show non-specific findings of arthropod bites,
although, rarely, biopsy may include a scabies mite or mite products, 4. The index case and any symptomatic family members will need
such as eggs and faecal material. to repeat the 5% permethrin cream treatment in one week from
initial application.
The adhesive tape test is another diagnostic technique and involves the
use of transparent tape (eg packing tape), which is firmly applied to a Careful identification followed by appropriate and thorough
lesion and then pulled off rapidly.11,14 The tape is subsequently applied treatment of patients, as well as their close physical contacts,
to a glass slide and examined under magnification. The rationale is that remain the cornerstone of scabies management.6 As scabies does
mites contained within the upper layers of the epidermis (ie the layers not always present with symptoms and clinical signs, all live-in
loosened by repetitive skin scratching) and those temporarily migrating family members and other close physical contacts should be
on the skin, should stick to the adhesive tape.11 This test is treated simultaneously, even in the absence of symptoms and
advantageous as it does not require specialised equipment. signs.6

19
CASE 3 check Infectious diseases

A range of agents, most of them topical, have been used to treat REFERENCES
scabies (Table 1).16 Topical preparations should be applied to the 1. Oakley A. Scabies. Hamilton, NZ: DermNet New Zealand, 2015. Available
entire body from the neck down, paying particular attention to the at www.dermnetnz.org/topics/scabies [Accessed 22 March 2017].
interdigital spaces, intergluteal cleft, umbilicus and subungual areas 2. Wong LC, Amega B, Barker R, et al. Factors supporting sustainability of a
(apply under the fingernails using a soft brush).4 In order to maximise community-based scabies control program. Australas J Dermatol 2002;43(4):274
exposure of the mite to the active drug, topical agents should be left 77.
on the skin for at least eight hours before washing.16 It is thus most 3. Wolff KJ, Johnson R, Saavedra A. Fitzpatricks colour atlas and synopsis of clinical
dermatology. 7th edn. New York: McGraw-Hill Medical, 2013.
practical to advise your patients to apply the solution in the evening 4. Bolognia JL, Jorizzo JL, Schaffer JV. Dermatology. 2012. London, Elsevier Health
and leave it overnight.4,16 To eradicate any newly hatched mites, a Sciences, 2012.
second application should be administered one to two weeks after the 5. Micali G, Lacarrubba F, Massimino D, Schwartz RA. Dermatoscopy: Alternative uses
first application.4,16 in daily clinical practice. J Am Acad Dermatol 2011;64(6):113546.
6. Heukelbach J, Feldmeier H. Scabies. Lancet 2006;367(9524):176774.
It should be noted that in certain populations, scabies can also occur 7. Expert Group for Dermatology. Dermatology: Insects and mites: Scabies. In: eTG
above the neck. This is most common in central and northern complete [Internet]. Melbourne: Therapeutic Guidelines Ltd, 2015. Available at
https://tgldcdp.tg.org.au/viewTopic?topicfile=insects-mites#toc_d1e47 [Accessed 9
Australia, and in infants and the elderly.17 In such individuals, topical February 2017].
formulations should also be applied to the face and hair, with care 8. Hengge UR, Currie BK, Jager G, Lupi O, Schwartz RA. Scabies: A ubiquitous
taken to avoid the eyes and mucous membranes.7 neglected skin disease. Lancet Infect Dis 2006;6(12):76979.
9. Gach JE, Heagerty A. Crusted scabies looking like psoriasis. Lancet
The risk of re-infestation via fomite transmission, although rare, can 2000;356(9230):650.
be minimised by laundering clothing, bed linens and towels at the time 10. Chosidow O. Clinical practices Scabies. N Engl J Med 2006;354(16):171827.
of treatment.4 Non-washable items can be stored in a hermetically 11. Walter B, Heukelbach J, Fengler G, Worth C, Hengge U, Feldmeier H. Comparison of
dermoscopy, skin scraping, and the adhesive tape test for the diagnosis of scabies in
sealed bag for one week.4 a resource-poor setting. Arch Dermatol 2011;147(4):46873.
Treatment of crusted scabies requires special consideration and we 12. Argenziano G, Fabbrocini G, Delfino M. Epiluminescence microscopy. A new
approach to in vivo detection of Sarcoptes scabiei. Arch Dermatol 1997;133(6):751
recommend specialist referral to a dermatologist or an infectious 53.
diseases physician. In general, treatment involves combination therapy 13. Albrecht J, Bigby M. Testing a test: Critical appraisal of tests for diagnosing scabies.
with repeated doses of ivermectin, frequent application of a topical Arch Dermatol 2011;147(4):49497.
scabicide and keratolytics. 14. Katsumata K. Simple method of detecting sarcoptes scabiei var hominis mites
among bedridden elderly patients suffering from severe scabies infestation using an
adhesive-tape. Intern Med 2006;45(14):85759.
ANSWER 6 15. Mounsey KE, McCarthy JS, Walton SF. Scratching the itch: new tools to advance
understanding of scabies. Trends Parasitol 2013;29(1):3542.
Re-infestation needs to be excluded. David needs to be closely examined 16. Currie BJ, McCarthy JS. Permethrin and ivermectin for scabies. N Engl J Med
for scabies burrows. If none are found, the likelihood is post-scabetic 2010;362(8):71725.
itch. This is because mites and mite products can still cause a significant 17. Kearns TM, Speare R, Cheng AC, et al. Impact of an ivermectin mass drug
administration on scabies prevalence in a remote Australian aboriginal community.
allergic response in the skin until they are shed. PLoS Negl Trop Dis 2015;9(10):e0004151.
18. Kearns T, Clucas D, Connors C, Currie BJ, Carapetis JR, Andrews RM. Clinic
Post-scabies itch can be expected to persist for up to four weeks after
attendances during the first 12 months of life for Aboriginal children in five remote
the end of successful scabicide therapy.10 This can be treated with communities of northern Australia. PLoS One 2013;8(3):e58231.
twice-daily administration of a medium-to-potent topical corticosteroid 19. Carapetis JR, Connors C, Yarmirr D, Krause V, Currie BJ. Success of a scabies
cream (eg mometasone furoate 1 mg/g cream) and emollient. When control program in an Australian Aboriginal community. Pediatric Infect Dis J
1997;16(5):49499.
symptoms persist beyond four weeks, treatment failure secondary to
poor compliance, resistance or re-infestation should be considered, as
well as diagnoses of nonscabetic origin.10

SCABIES IN AUSTRALIA
Scabies is endemic in many Aboriginal communities within
Australia.17,18 A review of clinic attendance by Aboriginal children of
remote northern Australia found that 69% of infants presented with
scabies in the first year of life.17 Scabies is associated with significant
morbidity.17 Secondary streptococcal infection can predispose children
to rheumatic heart disease and post-streptococcal glomerulonephritis,
which, worldwide, have the highest incidence in Australian Aboriginal
and Torres Strait Islander peoples.17 Mass drug administration
programs using topical permethrin to eradicate scabies and, more
recently, oral ivermectin, have been implemented with varying degrees
of success.2,18,19

20
check Infectious diseases CASE 4

QUESTION 2
CASE 4
What is the most likely diagnosis? What are the differential diagnoses?
ZOE HAS STOMACH CRAMPS What are the high risk diagnoses you need to consider?

Zoe, 26 years of age, is a teacher at the local high


school. She is your first patient for the day and presents
with a two-day history of frequent loose stools;
uncomfortable, crampy central abdominal pain; dull
headache; and feeling hot and cold. Apart from her
current symptoms, Zoe has no personal or family history
of significance. She is worried about passing it on to her
students, who are about to do their final exam.

QUESTION 1

How would you approach this consultation? What specific features of


the history and examination are you most interested in?
QUESTION 3

What investigations, if any, would you consider? What is your rationale


for this choice?

FURTHER INFORMATION
Zoe is fit and healthy. She has not been travelling or in contact with
anyone else with similar symptoms, Oh, except for Sally, my
neighbour, who had abdo cramps last week, but I think that was FURTHER INFORMATION
her periods. Sally has not been travelling. For the past 24 hours,
Zoe has had frequent small loose stools with a small amount of You assess Zoe as being mildly dehydrated and prescribe oral
blood and mucous. She has no significant medical history and rehydration solution, 23 L, to be taken in small volumes frequently
takes no medications. On examination, you note that Zoe looks over 24 hours. You advise her to eat if she feels hungry, and that
well, but uncomfortable, and is haemodynamically stable. She feels returning to her normal diet as soon as possible will help her
slightly thirsty. She has mild, generalised abdominal tenderness but recovery,1,2 but that she should be careful with foods containing high
no signs of peritonism and no abnormal masses. levels of fat or simple sugars. Large amounts of the latter, especially
soft drinks, sports drinks, cordials and fruit juice may increase the
osmotic load.2
You ask Zoe to stay home from work and return for review once the
results are available. You advise that she will not be able to return
to work until at least 48 hours after her stools return to normal.3
To prevent spread Zoe should be advised to follow simple hygiene
precautions, most importantly regular handwashing.4,5

21
CASE 4 check Infectious diseases

QUESTION 4 QUESTION 6

What is your management strategy while you await the results of the What are the serious complications associated with the more prevalent
stool sample? gastrointestinal infections in Australia?

FURTHER INFORMATION
FURTHER INFORMATION You see three other patients with symptoms similar to Zoes symptoms.
You are now concerned about whether these episodes of diarrhoea are
Your next patient, Zoes aunt, Anne, aged 75 years, comes to see linked and whether you need to contact the local public health unit (PHU).
you. She has weakness in both of her legs and has had increasing
difficulty walking upstairs over the past three days. Anne feels a You phone your local PHU and report your suspicion on the now four to
severe, deep ache in her legs when moving them, especially at five possibly linked cases of gastroenteritis, with one of the cases
night. She also notes pins and needles in her hands and feet. On working in an educational institution.
specific questioning, Anne tells you that she has not seen Zoe for
The PHU trace your initial contacts and find many more people in town
weeks and it was nice to catch up in the waiting room. She also
with similar symptoms. They investigate both cases, and local water
mentions that she had loose stools and mild fever about two to
and food sources in an attempt to identify the source. The other general
three weeks ago, which have since resolved.
practice in town has now also reported several cases.

QUESTION 5 Zoes stool result returns positive for Campylobacter. You are now
seeing many more people with the same symptoms, and those without
What is the most likely diagnosis in view of the associated symptoms but who are worried about getting it. You have had to
gastroenteritis in the preceding weeks? increase your hours to manage the extra load.
The evening news reports on large numbers of work absences due to
gastro sickness. One of the local primary schools has closed because
of the high number of staff affected. The stores in town have all sold
out of bottled water.

QUESTION 7

What are the requirements, if any, for notification of Campylobacter infection?

FURTHER INFORMATION
You consult a specialist for advice about Annes condition. The
specialist organises for Anne to be admitted to hospital for a lumbar
puncture, electromyography investigations and therapy.6

22
check Infectious diseases CASE 4

QUESTION 8 Universal infection-control precautions are needed throughout the consultation


and within the practice to avoid spread of any communicable disease.
What are the key components of the emergency health response to such an
outbreak? This includes community contacts and risk of spread within the
community. Zoe works in a school, so is in a higher risk occupation for
communicable disease spread. Certain diarrhoeagenic enteric pathogens are
notifiable, varying by state and territory (refer to Resources for doctors).
General practitioners (GPs) have a crucial role in surveillance and early
detection of infectious outbreaks and emerging infectious disease in their
local communities.
Clinical history should explore the patients presenting problem, with a focus
on the context in which it occurred. Questions should be asked to elicit
information about:
fluid intake and output, including:
vomiting, urine output and urine colour
stool characteristics onset, frequency, type, volume, presence of
blood (note: diarrhoea of the small bowel is typically watery, large
volume, associated with abdominal cramping, bloating and gas.14,15
Large bowel diarrhoea is associated with frequent regular small volume
bowel movements that may be painful with fever, and bloody or
CASE 4 ANSWERS
mucous stools)14
past medical history, including vaccinations and potential sources of
ANSWER 1 Salmonella infections (eg questionable food consumed, travel, unwell
contacts, animal exposure and occupation involving exposure to sources of
Diarrhoea is one of the top 30 presentations to Australian general practice, infection
with abdominal pain, diarrhoea and vomiting accounting for 2.7% of primary
reasons for presentations.7 There were more than 1.2 million hospitalisations medications
for acute gastroenteritis (AGE) in the nine years between 2001 and 2010.8 antibiotics are associated with Clostridium difficile infection
Diarrhoea has a reported prevalence of 6.4% in Australian communities, with
one in five seeking medical care. Children under five years of age have the proton pump inhibitors (PPI) increase the risk of infectious diarrhoea
highest prevalence.9 immunosuppressants increase the risk of cytomegalovirus (CMV) and
The introduction of rotavirus vaccine in the National Immunisation Program parasitic infections14
Schedule in 2007 has resulted in a considerable decrease in rotavirus and all-
absorption of some medications may be reduced (eg combined oral
cause gastroenteritis notifications, and hospital presentations in children
contraceptive pill)
younger than 5 years, with a 71% reduction in hospitalisation rates for
rotavirus gastroenteritis after three years of the program.7,8,10 It also appears medications more likely to cause adverse effects in the presence of
to have had a broader effect with a reduction in hospitalisations in the 519 dehydration include non-steroidal anti-inflammatory drugs (NSAIDs),
years age group as well.8 angiotensin converting enzyme inhibitors (ACEIs), angiotensin II
receptor blockers, diuretics, digoxin, warfarin, metformin, lithium
In Zoes case, the acute onset of abdominal pain, diarrhoea and fever could
be minor and self-limiting, but could also point to serious and potentially fatal comorbidities
causes.
pregnancy (risk of listeriosis increases 20-fold)14
A thorough history and examination are essential to establish:1113
epidemiological evidence including food history, travel, unwell contacts,
hydration status similar recent presentations.11,14
clinical severity Clinical examination may be entirely normal, but should include general
associated comorbidities and potential effect of gastroenteritis appearance, vital signs (eg heart rate, respiratory rate, blood pressure,
temperature), fluid status, body weight, and abdominal signs, including
medications and potential effect of gastroenteritis masses, distension, tenderness, signs of peritonism, bowel sounds.11,16
epidemiological evaluation Rectal examinations can assist in clarifying features of the stool including
blood and mucous.16
probability diagnosis and
Assessing hydration status of a patient is the key to appropriate rehydration
selection of appropriate candidates for faecal studies. strategies. Signs of severe dehydration include altered level of consciousness,

23
CASE 4 check Infectious diseases

intense thirst, dry mucus membranes, marked loss of skin turgor, In Zoes case, as there was blood in the stools, and as she is a school
haemodynamic instability, decreased/no urine output and increased urine teacher with a high risk of spreading any infectious disease to others, early
osmolality with ketones.13 In elderly patients, these signs and symptoms may diagnosis and management would be beneficial. It would be useful to perform
be absent, and dehydration may present atypically with increasing confusion, a stool culture and a faecal multiplex polymerase chain reaction (PCR).12,20
chest pain or falls.13
The PCR provides rapid results within 24 hours and increased sensitivity. It
currently detects the more common pathogens (ie Salmonella,
ANSWER 2 Campylobacter, Shigella, Yersinia enterocolitica, Aeromonas, Giardia,
The combination of fevers, cramping abdominal pain and bloody diarrhoea is Entamoeba histolytica, Dientamoeba, Blastocystis and Cryptosporidium).
suggestive of bacterial enteritis. In Australia and worldwide, diarrhoea is most However, molecular methods (PCR) may also detect asymptomatic infection
commonly caused by viral pathogens. In returned travellers, up to 90% of or non-viable organisms16 and have increased detection rates of rarely
diarrhoea is caused by bacteria.13,16 Zoe presents with mild dysentery. Visible pathogenic organisms such as Blastocystis hominis and Dientamoeba fragilis.
blood in the stool in febrile patients generally indicates infection due to
invasive pathogens such as Shigella, Salmonella and Campylobacter.11 Table 1. Causative pathogens and clinical features18,20,21
The main infectious differentials (for fever and bloody or mucoid stools) Pathogen Incubation period Clinical features
besides Salmonella, Shigella and Campylobacter include parasites
Salmonella Usually 1236 hours Headache, fever, abdominal
(Entamoeba histolytica), cytotoxic infection (C. difficile) or enterohaemorrhagic
with range 672 cramps, diarrhoea (+/ blood
Escherichia coli (EHEC) infection and, rarely, enteric viruses (CMV or hours or mucus), nausea
adenovirus).14 Other differentials include initial presentations of chronic
conditions, such as inflammatory bowel disease (ulcerative colitis, Crohns Shigella Usually one to three Fever, vomiting, abdominal
disease),14,16 or diarrhoea with haemorrhoids or periods.17 The rapidity of days, or up to seven cramps, diarrhoea (+/ blood
onset of severe abdominal pain means that ruling out an abdominal condition days or mucous)
requiring surgery, including ischaemic colitis, is essential.16 Campylobacter Usually two to five Fever, nausea, abdominal
In a patient with suspected gastroenteritis, clinical features may suggest the days, range of 110 cramps and diarrhoea (+/
days blood or mucous). Vomiting is
causative pathogen (Table 1). uncommon
Note: for acute diarrhoea, maintaining and correcting intravascular Shiga toxin Three to four days, Diarrhoea (+/ blood) and
fluid volume takes precedence over identifying a causative agent.11 For producing range of 210 days abdominal cramps. Little or no
most patients, such as Zoe, who do not have severe illness or high-risk Escherichia coli fever or vomiting
comorbidities, it is reasonable to make a clinical diagnosis and continue (STEC) 0157
expectant management.14
Yersinia Four to seven days Fever, abdominal pain and
cramps, diarrhoea (often blood
ANSWER 3 in kids), arthralgia in 50%
adults
Stool culture may be indicated if identification of potential bacterial pathogens
would alter management or suggest possible complications.16,18 In Australia, Clostridium Risk factors: recent Watery diarrhoea (+/ blood),
in 2011, less than one in five people presenting with diarrhoea were asked to difficile antibiotic use, long fever, anorexia, nausea,
submit a stool sample.19 Indications for stool culture include: hospital stay, elderly, abdominal pain
immunosuppression.
profuse watery diarrhoea with signs of dehydration
Entamoeba Days to weeks Often asymptomatic, or severe
frequent small volume stools with blood or mucus histolytica abdominal cramps, diarrhoea
(+/ blood or mucous), mild
bloody diarrhoea fever
temperature 38C
Such results need to be carefully interpreted in the clinical context to avoid
severe abdominal pain
unnecessary antimicrobial treatment. It should not be used in the absence
recent antibiotic use or hospitalisation13 of symptoms.22,23 Furthermore, PCR will not report on antimicrobial
sensitivities so subsequent stool culture is still needed to provide
older age (>70 years) or immunocompromised patient
susceptibility testing and for public health reasons.2
comorbidities that may be exacerbated by hypovolaemia
Collection of stool samples early in the illness is preferable as the number
symptoms persisting for more than a week
of pathogens in stool samples decreases with time. The specimen should
public health concerns (eg healthcare workers, food handlers, teachers, be stored in a sterile container and refrigerated to prevent overgrowth of
childcare workers or at the beginning of a pandemic).13,16,20 non-pathogenic organisms, and processed as soon as possible (within a

24
check Infectious diseases CASE 4

day or two of collection).13 Submission of a second sample increases the paraesthesia and pain that progresses proximally.28 About two-thirds of
detection rate to 99% of bacterial agents but is rarely required if PCR patients with GBS report respiratory or gastrointestinal symptoms in the
testing has also been requested.24 preceding three weeks, and 5089% experience nociceptive and/or
neuropathic pain with movement in the affected muscles that is worse at
Bacterial pathogens are generally excreted continuously, and so, a
night.6 Campylobacter jejuni is the most commonly recognised precipitant of
negative culture is usually not false.14 Stool culture results are usually
GBS and may trigger around 30% of all cases. Those subtypes with a slower
reported within three days and provide microbial sensitivity.25 Common
recovery and a higher residual disability risk are more likely to be preceded
viruses can be detected using highly sensitive and specific enzyme
by gastrointestinal symptoms.6 Although this is an uncommon condition,
immunoassay.25 More specific requests for the cause of bloody diarrhoea
early diagnosis can improve outcomes.6 Most people recover spontaneously,
in travellers could be considered for the detection of cysts, ova and
but given the unpredictable and potentially life-threatening and disabling
parasites with a possibility of E. histolytic or EHEC, as well as direct
nature of GBS, it is advisable to discuss Annes condition with a specialist.
testing for Shiga toxin and faecal lactoferrin.14,16
Routine laboratory tests are generally not required and would not be ANSWER 6
indicated for Zoe.3,25 If the patient is significantly dehydrated, serum
electrolytes, urea and creatinine may be indicated as a screen for renal Most commonly, gastroenteritis can cause volume depletion and electrolyte
dysfunction.14 disturbance (eg hypernatraemia, hyponatraemia, hypokalemia).29 Other
potential outcomes include acute renal failure (from severe volume
depletion) and sepsis.29 Other specific complications of bacterial enteritis
ANSWER 4
include reactive arthritis, cardiac complications (eg pericarditis, myocarditis)
Management of patients with acute diarrhoea begins with fluid repletion and and haemolytic uremic syndrome (from Shiga toxin-producing E. coli).4,16
nutrition maintenance. The high incidence of viral and self-limiting
gastroenteritis in Australia does not support empirical antibiotic treatment for ANSWER 7
uncomplicated illness1214 particularly with current concern about
antimicrobial resistance with inappropriate use of antibiotics.22 In a case In Victoria where you are working, gastroenteritis caused by Campylobacter
such as Zoes, where there is a mild presentation of bloody diarrhoea, is a notifiable disease.30,31
waiting for the results of the stool testing is justifiable.22 The purpose of Note: There is variation between states in notification requirements and so it
antibiotic therapy is to decrease duration, severity, excretion and spread of is worth checking locally. In Victoria Campylobacter infection is a group B
infection, or to prevent serious complications.13 In cases of severe bloody condition so requires completion of the Group B form to the Department of
diarrhoea, the use of empirical antibiotic therapy, such as azythromycin 500 Health and Human Services within five days. At time of publication
mg once daily for three days, or an oral fluoroquinolone, such as norfloxacin Campylobacter infection is not notifiable in NSW but this is likely to change
400 mg twice daily for three to five days, would be appropriate. in the near future to align with other states. In 201216, an estimated
Fluoroquinolone resistance is increasing, especially in Campylobacter.12,14 >18,000 cases of Campylobacter infections were reported annually across
Antimotility antidiarrhoeal agents such as loperamide have been shown to Australia.30 This number is likely to be an underestimation and the actual
reduce diarrhoea in randomised controlled trials, but should be avoided in number of cases is steadily increasing.31,32
children and the elderly because of potential adverse effects,10 such as toxic
magacolon. In a case with bloody diarrhoea, these would not be ANSWER 8
recommended unless antibiotics were also being taken, as they may prolong
GP surveillance role
the illness or increase its severity.4,5,7 Anti-emetics may be considered in
adults to reduce the severity of nausea and vomiting.6 GPs have a crucial role to play in early identification of infectious outbreaks
and emerging infectious disease, including those related to bioterrorism.17
Dietary aims in gastroenteritis are to return to as normal a diet as soon as
possible after rehydration.2 Although transient lactase deficiency is common GPs are spread through rural and urban communities and, with emergency
following gastroenteritis, a meta-analysis of clinical trials has indicated that a departments (EDs), are the frontline health contact with these communities.
lactose free diet is rarely necessary.1 A heightened awareness is required to differentiate an isolated presentation
from the start of a larger outbreak.
ANSWER 5 GPED interface
Anne might have developed Guillain-Barr syndrome (GBS). Reactive High-quality collaboration and communication between EDs and GPs are
arthritis and GBS are complications of Campylobacter gastroenteritis. The vital in such public health events. Real-time communication and information
peak incidence is in men aged 6575 years and women 7585 years.26 sharing between emergency health providers is required to ensure home-
The chance of developing GBS in the two months following symptomatic based care or hospital-based care is provided appropriately. Strong linkages
Campylobacter enteritis is <2/10,000 with a higher risk with Campylobacter and functional systems for business as usual for patients moving between
jejuni.27 primary care and emergency departments will maximise the chances of
GBS is a group of immunologically mediated polyneuropathies. The most having the best possible health response for a community during infectious
common form can present with weakness of the lower limbs with associated disease outbreaks.

25
CASE 4 check Infectious diseases

REFERENCES
CONCLUSION 1. Murphy M. Guidelines for managing acute gastroenteritis based on a systematic review of
published research. Arch Dis Child 1998;79:27984.
By the end of your first week, an alert comes through from the PHU, warning
2. ETG_Complete. Supportive management of acute gastroenteritis. Melbourne: Therapeutic
about the large number of patients presenting with gastroenteritis due to Guidelines Ltd, 2016.
Campylobacter and the suspicion that it is linked to the towns water supply, 3. Kelly A, Cheong E. How to treat: Paediatric gastroenteritis. Sydney: Australian Doctor, 2007.
which today tested positive for Campylobacter. The Mayor announced that 4. Juckett G. BMJ best practice Campylobacter infection. London: BMJ Best Practice, 2015.
the water is now being treated with chlorine, which should eradicate any 5. Ejemot RI, Ehiri JE, Meremikwu MM, et al. Hand washing for preventing diarrhoea. Cochrane
Database Syst Rev 2008;1:CD004265.
Campylobacter, but residents have been advised to boil all water until the
6. Walling AD, Dickson G. Guillain-Barre syndrome. Am Fam Physician 2013;87(3):19197.
final test results are available. Water tankers have been stationed throughout
7. Britt H, Miller G, Henderson J, et al. General practice activity in Australia 201415. Sydney:
the town. Sydney University Press, 2015.
8. Dey A, Wang H, Menzies R, Macartney K. Changes in hospitalisations for acute
The news that evening reports that there have been hundreds of gastroenteritis in Australia after the national rotavirus vaccination program. Med J Aust
presentations of gastroenteritis, including 22 requiring admission to hospital. 2012;197(8):45357.
All schools in the town have now closed as more than 50% of staff and 9. Scallan E, Majowicz SE, Hall G, et al. Prevalence of diarrhoea in the community in Australia,
children are absent with gastroenteritis. Canada, Ireland, and the United States. Int J Epidemiol 2005;34(2):45460.
10. Australian Technical Advisory Group on Immunisation. Part 4: Vaccine preventable diseases.
[This scenario has been loosely based on an actual outbreak.] In: Department of Health. The Australian immunisation handbook. 10th edn. Canberra: DoH,
2017.
11. World Gastroenterology Organisation. World Gastroenterology Organisation Global Guidelines
RESOURCES FOR PATIENTS Acute diarrhea in adults and children: a global perspective. Milwaukee, WI: World
Gastroenterology Organisation, 2012.
GBS CIDP Foundation International provides explanations about GBS, 12. Riddle MS, DuPont HL, Connor BA. ACG clinical guideline: Diagnosis, treatment, and
www.gbs-cidp.org prevention of acute diarrheal infections in adults. Am J Gastroenterol 2016;111(5):60222.
13. Acute gastroenteritis [Internet]. Therapeutic Guidelines Ltd, 2016. Available at www.tg.org.
GBS CIDP State based Groups exist in Australia including: au [Accessed 28 March 2017].
14. Wanke C, Calderwood SB, Bloom A. Approach to the adult with acute diarrhea in resource-
Queensland: www.gbsqld.com rich settings. Alphen aan den Rijn, Netherlands: UpToDate, 2016.
15. Wanke C. Small intestinal infections. Current opinion in gastroenterology 1994;10(1):5965.
New South Wales: www.gbs-cidp-nsw.org.au
16. Pawlowski S. BMJ best practice assessment of acute diarrhoea. London: BMJ Publishing
Group Ltd, 2016.
Victoria: www.ingroup.org.au
17. Dalton T. How to treat: Foodborne illness. Sydney: Australian Doctor, 2005.
Tasmania: www.connectingup.org 18. NSW Health. Foodborne-illness outbreak control guideline. Sydney: NSW Health, 2015.
Available at www.health.nsw.gov.au/Infectious/controlguideline/Pages/foodborne-illness.
aspx [Accessed 28 March 2017].
RESOURCES FOR DOCTORS 19. Australian Institute of Health and Welfare. Australian burden of disease study: Impact and
causes of illness and death in Australia 2011. Canberra: AIHW, 2016.
Notifiable diseases by jurisdictions: 20. Guerrant RL, Van Gilder T, Steiner TS, et al. Practice guidelines for the management of
infectious diarrhea. Clin Infect Dis 2001;32(3):33151.
Tasmania: www.dhhs.tas.gov.au/__data/assets/pdf_ 21. SA Health. Infectious diseases. Adelaide: SA Health, 2017. Available at www.sahealth.sa.
file/0003/53319/Guidelines_for_Notifying_Diseases_and_Food_ gov.au/wps/wcm/connect/Public+Content/SA+Health+Internet/Health+topics/Health+condi
Contaminants_FINAL_ISSUED.pdf tions+prevention+and+treatment/Infectious+diseases [Accessed 28 March 2017].
22. Spelman D, Jenny AW, Burgner DP. Australasian Society for Infectious Diseases: Low value
Victoria: www2.health.vic.gov.au/public-health/infectious-diseases/ interventions. Med J Aust 2017;206:28283.
notify-condition-now 23. Jones CA, David JS, Looke DF. Death from an untreatable infection may signal the start of
the post-antibiotic era. Med J Aust 2017;206:29293.
Western Australia: ww2.health.wa.gov.au/Articles/N_R/Notification- 24. Valenstein P, Pfaller M, Yungbluth M. The use and abuse of routine stool microbiology: A
of-infectious-diseases-and-related-conditions College of American Pathologists Q-probes study of 601 institutions. Arch Pathol Lab Med
1996;120(2):20611.
New South Wales: www.health.nsw.gov.au/Infectious/Pages/ 25. Hewison CJ, Heath CH, Ingram PR. Stool culture. Aust Fam Physician 2012;41(10):77579.
notification.aspx 26. Hughes RA, Charlton J, Latinovic R, Gulliford MC. No association between immunization and
Guillain-Barre syndrome in the United Kingdom, 1992 to 2000. Arch Intern Med
Queensland: www.health.qld.gov.au/clinical-practice/guidelines- 2006;166(12):130114.
27. TAM CC, Rodrigues LC, Petersen I, Islam A, Hayward A, OBrien SJ. Incidence of Guillain-
procedures/diseases-infection/notifiable-conditions/list
Barre syndrome among patients with Campylobacter infection: A general practice research
Australian Capital Territory: http://health.act.gov.au/sites/ database study. J Infect Dis 2006;194:9597.
28. Vriesendorp F. Guillain-Barre syndrome in adults: Clinical features and diagnosis. Alphen
default/files/Reporting_of_notifiable_conditions_code_of_ aan den Rijn, Netherlands: UpToDate, 2016.
practice_and_form_0.pdf 29. Brown K. BMJ best practice viral gastroenteritis. London: BMJ Best Practice, 2016.
30. Communicable Diseases Network Australia. National Communicable Diseases Surveillance
Northern Territory: https://health.nt.gov.au/professionals/centre-for- Report Fortnight 25, 2016. Summary notes for selected diseases 3 to 16 December
disease-control/cdc-programs-and-units/notifiable-diseases 2016. Canberra: Department of Health, 2016.
31. National Notifiable Diseases Surveillance System. Canberra: Department of Health Australian
National disease notification, www.health.gov.au/internet/main/ Government; 2017.
publishing.nsf/Content/cda-surveil-nndss-casedefs-distype.htm 32. Humphries RM, Linscott AJ. Laboratory diagnosis of bacterial gastroenteritis. Clin Microbiol
Rev 2015;28(1):331. connect/6701668047c17b259896da

26
check Infectious diseases CASE 5

QUESTION 3
CASE 5
What are your differential diagnoses?
MARY-JOY HAS AN ONGOING COUGH
Mary-Joy, 23 years of age, is from the Philippines. She
presents to your clinic with a five-week history of
cough. She has not had any shortness of breath, fevers
or night sweats. Mary-Joy notes that she has lost 3 kg
over the past two months but believes this is because
of stresses at work. You note on her past history that
Mary-Joy had a positive Mantoux test (19 mm) when
she first came to Australia as a nursing student. She
was clinically well and had a normal chest X-ray, but
was diagnosed with latent tuberculosis infection (LTBI).
At the time, she was preoccupied with moving house
and did not attend any further follow-up to address her
LTBI status.

FURTHER INFORMATION
Mary-Joy tells you that she was born in the Philippines and moved
QUESTION 1 to Australia two years ago. She has not travelled elsewhere,
What is your first priority in managing this presentation? currently works in a non-clinical role, and is not aware of any
known TB contacts. As an infant, she received the BCG vaccination
against TB.
She is otherwise well and is a non-smoker. On examination, she is
afebrile and observations are within normal limits. Her weight is 52
kg (55 kg, two months ago). Mary-Joy has no evidence of
respiratory distress. Her chest is clear and there is no palpable
lymphadenopathy.

QUESTION 4

How efficacious is the BCG vaccination against TB disease?

QUESTION 2

What information should you obtain on history? What physical


examination would you perform?

27
CASE 5 check Infectious diseases

QUESTION 5 QUESTION 7

What are risk factors for progression of LTBI to active TB? How would you interpret Mary-Joys chest X-ray?

FURTHER INFORMATION
QUESTION 6 Given the findings on the chest X-ray, you refer Mary-Joy to a TB
If you suspect that a patient has active pulmonary TB, what clinic. Although the sputum sample is negative for AFB, the TB unit
investigations are appropriate? requests polymerase chain reaction (PCR) testing as there is high
clinical and radiological suspicion of TB in Mary-Joys case. The PCR
results are positive for M. tuberculosis and indicate no rifampicin
resistance. Sputum cultures subsequently confirm M. tuberculosis.

QUESTION 8

How would you manage Mary-Joy? What is the treatment for pulmonary TB?

FURTHER INFORMATION
You collect a sputum sample from Mary-Joy, order a chest X-ray
(Figure 1) and refer her to a TB clinic.

Figure 1. Mary-Joys chest X-ray

QUESTION 9

What are potential side effects of Mary-Joys TB medications?

28
check Infectious diseases CASE 5

FURTHER INFORMATION
Mary-Joy is seen daily by TB nurses for DOTS. She has a monthly
CASE 5 ANSWERS
medical review at the TB clinic to review clinical progress, adjust
medications if required, and identify any new issues.
ANSWER 1
At three months she has no cough and has gained weight back to
baseline. She is tolerating her medications and experiences no side Tuberculosis (TB) is spread via airborne droplets. If active pulmonary
effects. Mary-Joy had no changes to her baseline visual acuity or disease is suspected, infection control measures are a priority at the
colour-vision testing (Ishihara test). A repeat chest X-ray shows clinic to reduce transmission to other patients and healthcare
minor residual left upper lobe findings. providers. Depending on local policies, this may include moving
patients away from crowded areas, asking the patient to wear a
On contact tracing, Mary-Joys three housemates, who are well, surgical mask, including when being transferred (eg to radiology), and
are identified as at-risk contacts and referred for screening. Mary- for the general practitioner (GP) to wear a N95 mask.
Joy is encouraged to talk with her family, who are in the
Philippines, to be aware of the signs and symptoms of TB, and to
ANSWER 2
consider being screened for LTBI.
Given Mary-Joys prolonged cough and history of known LTBI on
QUESTION 10 previous Mantoux tuberculin skin test and assessment, active
pulmonary TB is an important consideration. In a young person with
What is the purpose of contact tracing? What investigations are used? prolonged cough and weight loss, questions to assess for the
presence of TB include:1
history of known TB contacts (eg family, friends, occupational, others)
country of birth, travel history and age of arrival in Australia
characterising symptoms of cough, sputum production,
haemoptysis and pleuritic chest pain
constitutional symptoms of fevers, night sweats, weight loss and
lethargy.
Other information to elicit, to establish differential diagnoses include:
symptoms of wheeze, shortness of breath, rhinitis, sore throat, reflux
past medical and family history, including history of atopy
medications, known allergies and immunisations
social history, including smoking status, occupation and home
situation (eg children).
A full physical examination should be conducted. Specifically, this
would need to include examining vital signs, weight, respiratory
system and presence of lymphadenopathy.

ANSWER 3
In addition to active pulmonary TB, differential diagnoses for chronic
cough in Mary-Joys demographic are reactive airway diseases,
pertussis, atypical pneumonia, post-infectious cough, postnasal drip
and gastro-oesophageal reflux disease (GORD). Less common
differentials include fungal infections, non-TB mycobacterial infection,
non-acute presentation of melioidosis (in northern Australia),
bronchiectasis, pulmonary abscess and malignancy.1
TB is caused by Mycobacterium tuberculosis and spread via airborne
droplets. In most cases, infection (LTBI) is contained by the immune
system, without progression to active disease. One-third of the
worlds population is estimated to have LTBI. Approximately 10% of

29
CASE 5 check Infectious diseases

infected individuals develop active disease in their lifetime. Active TB ANSWER 4


can be pulmonary or, less commonly, extra-pulmonary, which can
The BCG vaccine is used mainly to protect against severe
include involvement of the lymph nodes, pleura, genitourinary tract, disseminated disease in young children (miliary and meningeal TB).
bones and joints.2 There is variable protection against adult pulmonary disease (080%).
Globally, the incidence of TB is highest across south-east Asia and It is not effective once a person is infected with M. tuberculosis.4
sub-Saharan Africa (Figure 2). TB remains one of the top 10
leading causes of death worldwide. The World Health ANSWER 5
Organizations (WHOs) African Region has the case fatality ratio Risk factors for progression to active TB include:5
(>20%), reflecting poor outcomes in the absence of timely TB
recent initial infection (within five years)
diagnosis, limited access to high-quality treatment, and regions of
high multidrug resistance.2 aged under five years and the elderly
In Australia, high-risk populations for TB infection include:3 comorbidities affecting immune function (eg untreated human
immunodeficiency virus [HIV] infection, end-stage renal disease,
recent migrants and returned travellers from high-incidence head and neck cancers, and haematological malignancies)
countries
other pulmonary comorbidities and smoking
contacts of active TB cases
medications such as tumour necrosis factor alpha (TNF-)
Aboriginal and Torres Strait Islander peoples in certain areas of the inhibitors, glucocorticoids, immune modulators used in organ
Northern Territory, Queensland and coastal New South Wales. transplantation and immune-compromising chemotherapy agents.
TB notification rates are much higher in people born overseas (19.5
per 100,000) than in those born in Australia (4.5 per 100,000); ANSWER 6
common countries of birth for TB cases in Australia include India, A flowchart showing the pathway for investigations when considering a
Vietnam, Philippines, China and Nepal.3 diagnosis of TB is shown in Figure 3.6

Figure 2. WHO estimated TB incidence rates, 2015

Reproduced with permission from the World Health Organization. Global tuberculosis report 2016. Geneva: WHO, 2016.

30
check Infectious diseases CASE 5

lymphadenopathy and there are minor changes within the right upper
Figure 3. Investigations when considering a diagnosis zone. These findings favour active TB.
of TB
Poorly defined consolidation favouring upper lobes (unilateral or
bilateral) is the most common chest X-ray finding for adult pulmonary
Consider TB TB. Other radiographic features may include volume loss, cavitation,
pleural effusion and hilar lymphadenopathy.8
Symptoms Note that in pulmonary TB, clinical symptoms, examination signs and
chest X-ray findings are variable. For patients with a chest X-ray
similar to Mary-Joys, there are often minimal symptoms on history
Latent Active
and a normal physical examination.

TST IGRA Chest X-ray and ANSWER 8


3 x sputum samples
More difficult Not if <2 The GP has an important role in considering TB as a diagnosis in high-
to interpret if years of risk patients and initiating prompt referral to TB services. Nationally,
previous age Microscopy TB is notifiable by clinicians and laboratories.
BCG vaccine
And TB treatment is usually supervised by the local TB service to provide
If TST or IGRA Culture Molecular directly observed therapy short course (DOTS). Mary-Joy is started on
consider
positive exclude assay standard short-course therapy of isoniazid, rifampicin, pyrazinamide
active TB AND and ethambutol. Pyridoxine (vitamin B6) is given to guard against
follow up peripheral neuropathy from isoniazid use.9 Ethambutol should be
Drug susceptibility
testing ceased when susceptibility testing verifies fully susceptible TB.9
Pyrazinamide is ceased after completing two months of treatment as
its benefit is only during the initial phase of treatment.9 Mary-Joy
Liaise with experts about approriate treatment should receive four further months of isoniazid and rifampicin (ie six
months total).
Reproduced with permission from The Royal Australian College of General Practitioners
from: Coulter C. Tuberculosis testing. Aust Fam Physician 2012;41(7):48992. She should also be given information on the importance of adhering to
the current combination of treatment medications for the full six
Initial investigations include a chest X-ray and three sputum samples, months to achieve cure and prevent selection of drug-resistant TB.
preferably early morning collections, for acid-fast bacilli (AFB) culture
and susceptibilities, in addition to standard microscopy, culture and ANSWER 9
sensitivity testing (MCS).3
Potential side effects of TB medications include:1,9
Sputum samples need to be collected in a well-ventilated place, away
from other people, and not in restroom facilities. For highly suspected Hepatitis, jaundice, nausea or vomiting (pyrazinamide, rifampicin,
clinical presentations of active TB, a spot sputum should be collected isoniazid)
and referral made to a hospital with respiratory isolation. In these Orange or red urine is expected from rifampicin
cases, patients should be kept in respiratory isolation (typically in a
hospital setting) until three sputum samples for AFB smears rule out Flu-like symptoms (rifampicin; increased risk with intermittent dosing)
active communicable TB.1,7 Tingling or numbness of hands or feet (isoniazid)
Molecular assays (PCR) for M. tuberculosis complex is a rapid Skin rash (all medications)
supplementary test that is more sensitive than AFB smear and less
sensitive than culture. PCR is routinely performed on new smear Colour vision/visual acuity impairment (ethambutol).
positive cases as it can confirm that the AFB seen are M. tuberculosis It is important for the patient and their regular GP to be aware of
complex and not non-tuberculosis mycobacteria. PCR also provides potential side effects and drug interactions. Patients should stop their
rapid information on rifampicin susceptibility to guide treatment. PCR
medication and seek review as soon as possible with the treating TB
may be requested in consultation with the TB unit and laboratory if
clinician if they experience concerning side effects.
clinical suspicion is high for active TB.3
Rifampicin is a cytochrome P450 inducer and can also reduce efficacy
ANSWER 7 of medications such as anticonvulsants, oral contraceptives and
warfarin.10 For these patients closer monitoring for drug interaction is
Mary-Joys chest X-ray shows patchy consolidation within the left required and alternative contraceptive options may need to be
upper zone. The left suprahilar region is prominent, suspicious of discussed.

31
CASE 5 check Infectious diseases

ANSWER 10 RESOURCES FOR PATIENTS


The purpose of contact tracing is to:3 NSW Healths TB fact sheets (including multicultural resources),
www.health.nsw.gov.au/Infectious/tuberculosis/Pages/tuberculosis-
identify other active TB cases: this may be the source of infection to
factsheets.aspx
the index case or a contact who has progressed to active TB
screen contacts for LTBI to opportunistically offer LTBI treatment RESOURCES FOR DOCTORS
after active TB is ruled out, to prevent progression to disease in the
future. Therapeutic Guidelines Mycobacterium infections,9 https://
tgldcdp.tg.org.au/viewTopic?topicfile=mycobacterial-infections
An algorithm for investigation of suspect active and LTBI is outlined in
Answer 4. CDNA National Guidelines for TB management,3 www.health.gov.
au/internet/main/publishing.nsf/Content/cdna-song-tuberculosis
For asymptomatic contacts, either a Mantoux test (ie TST) or interferon
gamma release assays (IGRA) can be used. Both test for a cellular Australian Family Physicians 2012 article on tuberculosis testing,6
immune response to previous mycobacterial antigen exposure. Overall, www.racgp.org.au/afp/2012/july/tuberculosis-testing
the two tests have comparable sensitivity and specificity for diagnosing State-based TB guidelines (eg NT),1 http://digitallibrary.health.nt.
LTBI. Refer to Table 1 for choice of test based on demographics. gov.au/prodjspui/bitstream/10137/696/4/TB%20Guidelines%20
May%202016.pdf
Table 1. Use of Mantoux (TST) or IGRA in diagnosing
LTBI6,11 REFERENCES
Demographics Preferred test Explanation 1. Centre for Disease Control. Guidelines for the control of tuberculosis in the Northern
Territory. Darwin: CDC, 2016. Available at http://hdl.handle.net/10137/696
General Mantoux (TST) Comparable sensitivity and specificity [Accessed 27 March 2017].
population OR IGRA Note: Sensitivity profiles of Mantoux 2. World Health Organization. Global tuberculosis report. Geneva: WHO, 2016 Available
(TST) and IGRA differ and the at www.who.int/tb/publications/global_report/en [Accessed 27 March 2017].
combination of both gives the highest 3. Communicable Diseases Network Australia. Tuberculosis (TB) CDNA national
guidelines for public health units Management of TB. Canberra: CDNA, 2015.
sensitivity Available at www.health.gov.au/internet/main/publishing.nsf/content/cdna-song-
tuberculosis [Accessed 27 March 2017].
Children <5 Mantoux (TST) IGRA is more likely to be false
4. National tuberculosis Advisory Committee. The BCG vaccine: information and
years negative in young children than adults recommendations for use in Australia. Communicable Disease Intelligence
Previous BCG IGRA Interpretation of Mantoux (TST) 2006;30(1): E110-115.
5. Getahun H, Matteelli A, Chaisson RE, et al. Latent Mycobacterium tuberculosis
vaccination is complicated by previous BCG
infection. N Eng J Med 2015;372(22):212735.
vaccination
6. Coulter C. Tuberculosis testing. Aust Fam Physician 2012;41(7):48992.
Hard to reach IGRA may be more suitable for 7. Chris Coulter, National Tuberculosis Advisor Committee. Infection control guidelines
populations (eg patients who are unlikely to attend a for the management of patients with suspected or confirmed pulmonary tuberculosis
in healthcare settings. Communicable Diseases Intelligence 2016;40(3):E36066.
homeless) second visit for Mantoux (TST) reading
8. Burrill J, Williams CJ, Bain G, et al. Tuberculosis: A radiologic review. RadioGraphics
2007;27(5):125573.
9. Therapeutic Guidelines Limited. Mycobacterial infections [Internet]. eTG Complete.
Should LTBI be confirmed, an individualised discussion will take place, Melbourne: eTG, 2016. Available at https://tgldcdp.tg.org.au/viewTopic?topicfile=my
weighing up the likelihood of reactivation with the risks and benefits of cobacterial-infections [Accessed 27 March 2017].
treatment. Therapeutic Guidelines recommends isoniazid for six to 10. Martin J, Fay M. Cytochrome P450 drug interactions: are they clinically relevant?
nine months as first-line treatment for LTBI.9 However, LTBI treatment Aust Prescrib 2001;24(1).
11. National Tuberculosis Advisor CommitteePosition statement on interferon- release
guidelines differ across jurisdictions, so adherence to the relevant local
assays in the detection of latent tuberculosis infection. Communicable Disease
guidelines is recommended.1 Intelligence 2012; 36(1): E125-131.
12. Ralph A, Kelly P, Krause V. Whats new in TB? Aust Fam Physician 2009;38(8):578
85.
CONCLUSION
While specialised TB units are primarily responsible for the
management of TB patients, GPs have an important role in the
diagnosis and ongoing care. This includes timely referral of patients to
TB services, initiating infection control, being aware of and monitoring
for side effects and drug interactions, managing comorbidities (eg poor
nutrition, smoking cessation), and providing ongoing support in the
patients social setting to complete curative treatment.12

32
check Infectious diseases MULTIPLE CHOICE QUESTIONS
MULTIPLE CHOICE QUESTIONS

B. polymerase chain reaction (PCR) for antigen detection


ACTIVITY ID: 96529
C. enzyme-linked immunosorbent assay (ELISA) for antibody
INFECTIOUS DISEASES detection

This unit of check is approved for six Category 2 points D. intradermal skin test.
in the RACGP QI&CPD program. The expected time to
complete this activity is three hours and consists of: CASE 2 ROSLYN
reading and completing the questions for each Roslyn, 30 years of age, presents with blisters around the genital
case study and anal areas. About a week ago, she noticed some itchy bumps
you can do this on hard copy or by logging on to in these areas but the itchiness has become more severe and
the gplearning website, http://gplearning.racgp. particularly painful when urinating. Roslyn has never had any
org.au genital or urinary tract infections, but is worried that she may have
answering the following multiple choice questions contracted an infection after having unprotected sex two weeks
(MCQs) by logging on to the gplearning website, ago with a man she met at a nightclub. Physical examination
http://gplearning.racgp.org.au reveals some blisters and genital ulceration. You take samples to
you must score 80% before you can mark the be tested for sexually transmissible infections (STIs). Testing
activity as Complete confirms Herpes simplex 2 (HSV-2) infection and you prescribe
antiviral treatment.
completing the online evaluation form.
You can only qualify for QI&CPD points by completing
QUESTION 3
the MCQs online; we cannot process hard copy
answers. Which of the following is currently recommended for treatment of a
first episode of genital herpes?
If you have any technical issues accessing this
activity online, please contact the gplearning A. Acyclovir 400 mg orally every eight hours for five days
helpdesk on 1800 284 789.
B. Acyclovir 400 mg orally plus topical application every eight hours
If you are not an RACGP member and would like to for five days
access the check program, please contact the
gplearning helpdesk on 1800 284 789 to purchase C. Valacyclovir 500 mg once daily for five days
access to the program. D. Any of the above options

QUESTION 4
CASE 1 MAISIE What advise can you give Roslyn about HSV-2 infection?
Maisie is 18 years of age and presents with a one-week history of A. Disease activity and viral shedding remain high in the first 1224
severe itching on her hands and wrists. From your examination and months after infection.
Maisies history, you consider a diagnosis of scabies.
B. In most patients, recurrences after a symptomatic first episode are
infrequent (less than one per year).
QUESTION 1
C. Sexual contact should be avoided during a symptomatic episode;
Which of the following symptoms or signs in a patient of Maisies age however, transmission can occur at other times.
are suggestive of scabies?
D. All of the above.
A. Itching is relieved by a hot bath or shower
B. Itching localised to the hands and wrists CASE 3 IVAN AND ERICA

C. Presence of asymmetrical lesions on the palms of the hands Ivan, a school teacher aged 35 years, emigrated from South Africa
with his wife, Erica, 12 months ago. He comes to see you
D. Widespread and intense pruritus that spares the head and neck complaining of a cough that has persisted for six weeks. He tells you
that he has also been feeling run down lately and has lost some
QUESTION 2 weight. He has no significant medical history, no allergies, is not on
any medications, and does not smoke. Ivan cannot remember if he
At present, a diagnosis of scabies is best made in practice via:
had a BCG vaccination, but his medical examination prior to
A. light microscopic examination of skin scrapings of burrows relocating to Australia was clear.

33
MULTIPLE CHOICE QUESTION check Infectious diseases

QUESTION 5 D. norfloxacin 400 mg twice daily for three days.


Which of the following warrants consideration of tuberculosis (TB) as
the diagnosis? CASE 5 INFLUENZA PANDEMIC

A. Ivans country of origin As you are preparing to leave work for the day, your practice receives
an alert from the Department of Health that new strain of influenza A
B. Persistent cough virus has been identified in a rapidly spreading outbreak in California.
C. Weight loss Over the next weeks, similar cases are reported in several other
countries and the World Health Organization (WHO) declares that a
D. All of the above pandemic is underway. So far the new viral strain has not been
detected in Australia.
FURTHER INFORMATION
Further investigations confirm that Ivan has active TB and you refer him QUESTION 9
to a TB clinic for treatment. Erica is well and has not developed any At this stage, the response in Australia to this alert would be:
symptoms. She had a BCG vaccination when she was 13 years of age.
A. Preparedness
QUESTION 6 B. Respond Standby
Which of the following options is correct with regards to screening C. Respond Initial action
Erica for latent TB infection (LTBI)?
D. Respond Targeted action
A. Erica should have a chest X-ray.
B. Erica should have a Mantoux test (TST) or interferon gamma FURTHER INFORMATION
release assay (IGRA). Three months after the initial alert, thousands of cases of the new
C. A sputum sample from Erica should be obtained for polymerase strain of influenza are being seen in general practices in Australia. The
chain reaction (PCR) testing. pandemic stage is now in Response Targeted action.
D. Erica does not need to be screened as she has had a BCG vaccination.
QUESTION 10
CASE 4 REG At this stage of the pandemic, the priority in managing patients
presenting with influenza-like illness is:
Reg, 58 years of age, is a chef at a local caf. He comes to see you
because he has been having headaches, abdominal cramps, nausea and A. obtaining epidemiological evidence of contact with confirmed
diarrhoea for the past three days. He has not noticed any blood in the cases of the new strain
stools. He has had no appetite since the onset of his symptoms, but has
B. collecting viral samples for further surveillance purposes
been feeling very thirsty. He has a temperature of 37.5C. Your provisional
diagnosis is gastroenteritis and you are concerned about bacterial enteritis. C. identifying those at risk of complications dependent on advice from
the Department of Health and public health unit, as management
QUESTION 7 may differ in each pandemic

In Regs case you would consider ordering a stool culture to confirm D. moving back to business as usual and revising plans for the next
the diagnosis and identify the causative agent because of pandemic.

A. the duration of Regs symptoms


B. a possible public health concern
C. Regs age
D. All of the above reasons

QUESTION 8
The most appropriate management for Reg is:
A. fluid repletion with an oral rehydration solution
B. fluid repletion and nutritional maintenance with fruit juice
C. azithromycin 500 mg once daily for three days

34
GP17
patien
n of t-c
tio

The founda

en
tred care
gp
RACGP CONFERENCE en

G
ic

e
Sydney + 2628 October 2017 + era
l prac
t

GP17: lets discuss the issues


Every week, every month and every year, you are a part of the health journey of your patients.
You are their specialist in life guiding them through each of their medical concerns. The program for
GP17 has been designed for you covering a rich and diverse range of topics relevant to you including:

+ Care of older people and their health issues


+ Children and young peoples health
+ Womens health and pregnancy
+ Travel medicine
+ Eye medicine
+ Musculoskeletal and sports medicine; as well as many more

GP17 not only provides CPD points across Categories 1 and 2 but it will also help you to fulfil some of
your evidence requirements for planning, learning and need (PLAN).

Wed love to see you there! Visit gp17.com.au to find out more or to register.

ICC Sydney
4807

gp17.com.au #GP17sydney

Vous aimerez peut-être aussi